Skip to content
🤔 AI effective tips collection 🧠
❓:Task B-3.4. In the plane, 100 concentric circles are drawn with radii 1 mathrm{~cm}, 2 mathrm{~cm}, 3 mathrm{~cm}, ldots, 100 mathrm{~cm}. The circle with a radius of 1 mathrm{~cm} is painted red, and all other areas bounded by consecutive circles are painted either red or green, but in such a way that adjacent areas are painted different colors. Determine the ratio of the area of all green regions to the area of all red regions.

🧠:[<think>Okay, so there's this problem with 100 concentric circles, right? Their radii go from 1 cm up to 100 cm, each increasing by 1 cm. The innermost circle, the one with radius 1 cm, is painted red. Then, the areas between consecutive circles (these are called annular regions, I think) are alternately colored red and green. The key thing is that adjacent areas can't be the same color. I need to find the ratio of the total green area to the total red area. Hmm.Let me visualize this. The first circle, radius 1, is red. Then the area between radius 1 and 2 is green, because it's adjacent to the red circle. Then the area between 2 and 3 would be red again, since it's adjacent to the green one. So the colors alternate: red, green, red, green, etc. So starting from the center, the first circle is red, the first annulus (1-2 cm) is green, the next annulus (2-3 cm) is red, and so on, up to the 100th circle.Wait, but hold on. The problem says "all other areas bounded by consecutive circles are painted either red or green, but adjacent areas are different colors." So the innermost circle is red, then the first annulus (1-2) must be green, next (2-3) red, etc. So the coloring alternates starting with red at the center. Therefore, the regions (circle and annuli) alternate between red and green. So regions 1 (radius 0-1) is red, 1-2 is green, 2-3 is red, 3-4 is green, ..., 99-100 is red or green depending on parity.Since there are 100 circles, the regions are from radius 0-1, 1-2, ..., 99-100. So total 100 regions. The first one (0-1) is red, then the next (1-2) green, alternating. So the color of the nth annulus (between n-1 and n cm) depends on whether n is odd or even. Wait, actually, the regions are between k-1 and k cm for k from 1 to 100. But the first region is 0-1, but radius starts at 1. Wait, maybe the first circle is radius 1, so perhaps the regions are 1-2, 2-3, ..., 99-100. Wait, the problem states "100 concentric circles are drawn with radii 1 cm, 2 cm, ..., 100 cm." Then the circle with radius 1 cm is painted red. The areas bounded by consecutive circles are painted red or green, with adjacent areas different colors.So, the first circle (radius 1) is a disk of radius 1, painted red. Then the area between radius 1 and 2 is an annulus; since it's adjacent to the red disk, it must be green. Then the area between 2 and 3 must be red, adjacent to the green annulus. Continuing in this way, alternating colors. So the regions alternate starting with red for the first disk, then green, red, etc. So the color of each annulus (between n and n+1 cm) is green if n is odd (since the previous annulus, n-1 to n, would be red when n is even), wait no. Wait, the first region is the disk (radius 1), red. Then the annulus from 1-2 is green. Then 2-3 is red. 3-4 green, etc. So each annulus from k-1 to k cm: for k=1, it's the disk, red; k=2, annulus 1-2, green; k=3, annulus 2-3, red; k=4, annulus 3-4, green; and so on. So the color alternates starting with red for k=1.Therefore, for each annulus between radius (k-1) and k cm, where k ranges from 1 to 100. But since the radii start at 1 cm, the first region is radius 0-1? Wait, but the first circle is radius 1, which is a disk. Wait, maybe the confusion is here: the problem says "the circle with radius 1 cm is painted red". A circle is just the circumference, but in context, when they say it's painted red, they probably mean the disk of radius 1 cm is red. Then the area between 1 cm and 2 cm (the annulus) is another region, which must be green. Then between 2 cm and 3 cm is red, etc. So the regions are:- Disk: radius 0-1 cm: red (but wait, the circle with radius 1 cm is painted red. If it's a circle, it's just the circumference, but maybe they mean the disk. The problem says "areas bounded by consecutive circles". So the circles are the boundaries, so the regions between them are annular regions. But the innermost region would be the disk inside the first circle. So maybe the regions are:1. Inside radius 1 cm: disk, red.2. Between 1 cm and 2 cm: annulus, green.3. Between 2 cm and 3 cm: annulus, red....100. Between 99 cm and 100 cm: annulus, color depending on parity.So there are 100 regions: 1 disk and 99 annuli. Wait, but 100 concentric circles would create 100 regions? Wait, n circles divide the plane into n regions if they are concentric. Wait, no. If you have n concentric circles, they divide the plane into n+1 regions: the innermost disk, then n-1 annuli, and the outer region. But in this problem, it says "100 concentric circles... The circle with a radius of 1 cm is painted red, and all other areas bounded by consecutive circles are painted either red or green". So maybe the 100 circles create 100 annular regions? Wait, radii 1,2,...,100 cm. So the regions are:- Inside the first circle (radius 1): disk.- Between 1 and 2: annulus.- Between 2 and 3: annulus....- Between 99 and 100: annulus.So that's 100 regions: 1 disk and 99 annuli? Wait, 100 circles would create 100 regions? Wait, normally, n concentric circles create n+1 regions: the innermost disk and n-1 annuli and the outer region. But here the circles are up to radius 100 cm, so the outer region is beyond 100 cm. But the problem says "all other areas bounded by consecutive circles". So maybe they are considering only the regions between the circles, excluding the outer area beyond 100 cm. Then, the number of regions bounded by consecutive circles is 99: between 1-2, 2-3, ..., 99-100. But the innermost region (the disk of radius 1) is also considered. So total regions: 1 disk + 99 annuli = 100 regions. That's probably right since there are 100 circles, each subsequent one larger, creating 100 regions (the first is the disk, then each annulus between k and k+1 for k from 1 to 99).But the problem states: "the circle with radius 1 cm is painted red, and all other areas bounded by consecutive circles are painted either red or green". So the innermost circle (disk) is red, and the rest of the annular regions (from 1-2 up to 99-100) are colored red or green alternately. Wait, but the problem says "adjacent areas are painted different colors". So the disk (0-1) is red. Then the first annulus (1-2) must be green. Then the next annulus (2-3) must be red, etc. So the colors alternate starting with red for the disk. Therefore, all even-numbered annuli (if we number the disk as region 1, annulus 1-2 as region 2, etc.) would be green, red, green, etc. Wait, let's see:Regions:1. Disk (0-1 cm): red2. Annulus (1-2 cm): green3. Annulus (2-3 cm): red4. Annulus (3-4 cm): green...100. Annulus (99-100 cm): color depends on whether 100 is even or odd.Since 100 is even, the 100th region (annulus 99-100) would be green? Wait, region 1 is red, region 2 is green, region 3 red, ..., region 100: if 100 is even, then green. Wait, starting from the first region (disk) as red, the parity of the region number determines the color. Odd-numbered regions are red, even-numbered are green. So region 1: red, 2: green, 3: red, ..., 100: green (since 100 is even). Therefore, in total, how many red regions and green regions?Total regions: 100. Regions 1,3,5,...,99 are red (total of 50), and regions 2,4,6,...,100 are green (total of 50). Wait, but the first region is the disk, and the rest are annuli. So there's 1 red disk, then 49 red annuli and 50 green annuli? Wait, let's count.Regions 1 to 100:Red regions: 1,3,5,...,99. Since 99 is the 99th region, which is odd. So from 1 to 99 odd, that's 50 regions (since (99 - 1)/2 +1 = 50). Wait, 1,3,...,99: how many terms? It's (99 - 1)/2 +1 = 50. So 50 red regions: 1 disk and 49 annuli. Then green regions: 50 annuli. Wait, 50 red regions (1 disk, 49 annuli) and 50 green regions (all annuli). Therefore, total red area is the area of the disk plus the areas of 49 annuli, and green area is the areas of 50 annuli.Now, we need to compute the total red area and total green area, then find their ratio.First, let's recall the area of a circle is πr². The area of an annulus between radius (k-1) and k is πk² - π(k-1)² = π(2k -1).Wait, let's check that. The area between radius n-1 and n is πn² - π(n-1)² = π(n² - (n-1)²) = π(n² - (n² - 2n +1)) = π(2n -1). So yes, the area of the annulus between n-1 and n is π(2n -1). Therefore, each annulus from k-1 to k has area π(2k -1). However, for the first region, the disk of radius 1, its area is π(1)² = π. So the disk area is π, and each subsequent annulus (k from 2 to 100) has area π(2k -1). Wait, hold on. Wait, the annulus between 1 and 2 cm is k=2, so area π(2*2 -1) = π(3). Similarly, the disk (k=1) is area π(1)^2 = π. So:- Region 1 (disk): area π(1)^2 = π.- Region 2 (annulus 1-2): area π(2^2 -1^2) = π(4 -1) = 3π.- Region 3 (annulus 2-3): area π(3^2 -2^2) = π(9 -4) = 5π....- Region 100 (annulus 99-100): area π(100^2 -99^2) = π(10000 -9801) = π(199) = 199π.So each annulus k (for k from 1 to 100) has an area of (2k -1)π. But note that for k=1, it's the disk, which is area π, and indeed 2*1 -1 =1, so 1π. Then for annulus k=2, it's 3π, which is 2*2 -1=3, etc. So each region k (disk or annulus) has area (2k -1)π. Therefore, the total area of all regions is sum_{k=1}^{100} (2k -1)π = π sum_{k=1}^{100} (2k -1). The sum of the first n odd numbers is n², so sum_{k=1}^{100} (2k -1) = 100² = 10,000. So total area is 10,000π. That makes sense since the largest circle has radius 100 cm, so area π(100)^2 =10,000π. So all regions combined make up the entire disk of radius 100 cm.Now, the red regions are the ones where k is odd (1,3,5,...,99). Each red region k has area (2k -1)π. Similarly, green regions are k even (2,4,...,100), each with area (2k -1)π.Therefore, total red area is sum_{k=1,3,5,...,99} (2k -1)π. Similarly, total green area is sum_{k=2,4,6,...,100} (2k -1)π.So let's compute these sums.First, for red regions: k =1,3,5,...,99. There are 50 terms (since from 1 to 99 with step 2). Each term is (2k -1)π. Let's compute the sum.Let me denote S_red = sum_{m=1}^{50} (2*(2m -1) -1)π. Wait, because if we let m be the index for red regions, where k=2m-1 for m=1 to 50. Then:Each term is 2k -1 = 2*(2m -1) -1 =4m -2 -1=4m -3.Wait, that seems complicated. Maybe better to compute the sum directly.Alternatively, note that each annulus (including the disk) has area (2k -1)π. So for red regions (k odd), the areas are:k=1: 1πk=3: 5πk=5:9π...k=99: (2*99 -1)π =197πSo this is an arithmetic sequence where the first term a1=1π, common difference d=4π (since 5π -1π=4π, 9π -5π=4π, etc.). Wait, is that right?Wait, let's check:For red regions:k=1: term is 1πk=3: term is 2*3 -1=5πk=5: 2*5 -1=9πk=7:13πYes, so the difference between consecutive red regions is 4π. So starting at 1π, each subsequent red annulus is 4π more. But wait, how many terms?Since k goes from 1 to 99 odd, which is 50 terms. So the sum of an arithmetic series is n/2*(2a1 + (n-1)d).Here, n=50, a1=1π, d=4π.So S_red = 50/2 * [2*1π + (50 -1)*4π] =25*[2π +49*4π] =25*[2π +196π]=25*198π=4950π.Similarly, the green regions are k=2,4,6,...,100. Each green annulus has area (2k -1)π. Let's compute S_green.For k=2: (2*2 -1)π=3πk=4: (2*4 -1)π=7πk=6:11π...k=100: (2*100 -1)π=199πSo this is also an arithmetic sequence. First term a1=3π, common difference d=4π (7π -3π=4π, 11π -7π=4π, etc.), number of terms n=50.Therefore, S_green =50/2*[2*3π + (50 -1)*4π]=25*[6π +196π]=25*202π=5050π.Therefore, total red area is 4950π, total green area is5050π.Thus, the ratio of green to red is5050π /4950π=5050/4950=505/495=101/99≈1.0202...But let's check that again. Wait, when computing S_red:Wait, the red regions:k=1:1πk=3:5πk=5:9π...k=99:197πSo the sequence is 1,5,9,...,197π. The number of terms is 50. The first term a1=1, last term l=197. Sum= n*(a1 + l)/2=50*(1 +197)/2=50*198/2=50*99=4950. So S_red=4950π.Similarly, green regions:k=2:3πk=4:7πk=6:11π...k=100:199πSequence is 3,7,11,...,199π. Number of terms=50. First term a1=3, last term l=199. Sum=50*(3 +199)/2=50*202/2=50*101=5050. So S_green=5050π.Therefore, ratio is 5050π /4950π=5050/4950=505/495=101/99.Simplify 101/99. 101 is a prime number, so the ratio is 101:99.Therefore, the ratio of green area to red area is 101/99.Wait, but let me verify once more. Total area is 4950π +5050π=10,000π, which matches the area of the largest circle, π*(100)^2=10,000π. So that checks out.Therefore, the answer should be 101/99.But wait, let me think again. The red regions start with the disk (area π) and then alternate. The green regions are all annuli. The total red area is 4950π, which is less than green's 5050π. So the ratio is green:red=5050:4950=101:99. Yes.Therefore, the ratio is 101/99.Final AnswerThe ratio of the area of all green regions to the area of all red regions is boxed{dfrac{101}{99}}.</think>The problem involves 100 concentric circles with radii from 1 cm to 100 cm. The innermost circle (radius 1 cm) is painted red, and the areas between consecutive circles are alternately colored red and green such that adjacent areas are different colors. We need to determine the ratio of the total green area to the total red area.1. Coloring Pattern: - The innermost circle (radius 1 cm) is red. - The areas between consecutive circles (annuli) alternate between green and red starting with green for the annulus between 1 cm and 2 cm.2. Regions and Areas: - Each annulus between radius ( k-1 ) cm and ( k ) cm has an area of ( pi(2k - 1) ). - The innermost circle (radius 1 cm) has an area of ( pi ).3. Summing the Areas: - Red Regions: These include the innermost circle and every alternate annulus starting from radius 2 cm. The areas are ( pi, 5pi, 9pi, ldots, 197pi ). This is an arithmetic sequence with the first term ( a_1 = pi ), common difference ( d = 4pi ), and 50 terms. - Sum of red areas: ( S_{text{red}} = frac{50}{2} times [2pi + (50 - 1) times 4pi] = 25 times [2pi + 196pi] = 25 times 198pi = 4950pi ). - Green Regions: These include every alternate annulus starting from the annulus between 1 cm and 2 cm. The areas are ( 3pi, 7pi, 11pi, ldots, 199pi ). This is an arithmetic sequence with the first term ( a_1 = 3pi ), common difference ( d = 4pi ), and 50 terms. - Sum of green areas: ( S_{text{green}} = frac{50}{2} times [2 times 3pi + (50 - 1) times 4pi] = 25 times [6pi + 196pi] = 25 times 202pi = 5050pi ).4. Ratio Calculation: - The ratio of the total green area to the total red area is ( frac{5050pi}{4950pi} = frac{5050}{4950} = frac{101}{99} ).Thus, the ratio of the area of all green regions to the area of all red regions is boxed{dfrac{101}{99}}.,<think>Okay, so I have this problem here with 100 concentric circles, each with radii from 1 cm up to 100 cm. The innermost circle, which is radius 1 cm, is painted red. Then the areas between consecutive circles (so like the area between radius 1 and 2, 2 and 3, etc.) are painted either red or green. The key thing is that adjacent areas have different colors. I need to find the ratio of the total green area to the total red area. Hmm, let me think.First, let's visualize this. We have 100 circles, so there are 99 annular regions between them. The first circle (radius 1 cm) is red. Then the next region, between 1 cm and 2 cm, must be a different color from red, so green. Then the area between 2 cm and 3 cm must be different from green, so red again, and so on. So it's alternating colors: red, green, red, green, etc. Since the first region (the innermost circle) is red, the pattern would be red, green, red, green... all the way out to the 100th circle.Wait, but the problem says "all other areas bounded by consecutive circles" are painted either red or green. So the innermost circle is red, and then each subsequent annulus alternates. So that's correct. So the coloring alternates starting with red at the center. So regions: 1st circle (radius 1) is red, then annulus from 1-2 is green, 2-3 is red, 3-4 is green, etc., up to 99-100. So total regions: 100 circles, but the regions between them are 99. Wait, but the innermost circle is the first region, and then each annulus from 1-2, 2-3, ..., 99-100. So actually, total regions are 100? Wait, no. Wait, if there are 100 circles with radii 1 to 100, then the regions are: the disk of radius 1 (area 1), then the annulus between 1 and 2 (area π(2² -1²)=3π), then between 2 and 3 (area 5π), etc. So the number of regions is equal to the number of circles, which is 100. Wait, but between n circles, you have n-1 regions. Wait, no. Wait, if you have 100 circles, then you have 100 boundaries at 1, 2, ..., 100 cm. The regions are: the innermost disk (radius 1), then each annulus from radius k to k+1 for k from 1 to 99. So total regions: 1 (disk) + 99 (annuli) = 100 regions. Wait, but the problem says "the circle with radius of 1 cm is painted red, and all other areas bounded by consecutive circles are painted either red or green". Hmm, so the innermost circle (radius 1) is red. The other regions (from 1-2, 2-3, ..., 99-100) are painted red or green such that adjacent areas are different colors. So the regions adjacent to the innermost circle (which is red) must be green. Then the next one must be red, and so on. Therefore, the coloring alternates starting with red for the innermost circle. So the regions alternate red, green, red, green, etc. So the 1 cm circle is red, 1-2 cm annulus is green, 2-3 cm is red, 3-4 cm is green, ..., 99-100 cm is green if 99th region? Wait, let's check the parity. Starting with red (1st region), then the next is green (2nd region), red (3rd), green (4th), ..., so the nth region (where n goes from 1 to 100) is red if n is odd, green if n is even. Wait, but the regions: the first region (radius 1) is 1, then regions 2 to 100 are the annuli from 1-2 up to 99-100. Wait, hold on. Let's clarify:Total regions:1. Disk: radius 0 to 1 (but in the problem, the circles start at radius 1. Wait, hold on. Wait, the problem says "100 concentric circles are drawn with radii 1 cm, 2 cm, ..., 100 cm". So the first circle is at radius 1 cm. Then the areas bounded by consecutive circles: so between radius 1 and 2 is the first annulus, 2 and 3 is the next, etc. So the regions are from 1-2, 2-3, ..., 99-100. But the innermost region is the disk of radius 1, right? Because the first circle is radius 1. So the problem states: "the circle with a radius of 1 cm is painted red", which is the innermost disk. Then "all other areas bounded by consecutive circles" i.e., the annuli from 1-2 up to 99-100 are painted red or green with adjacent areas being different colors. So the disk (radius 1) is red. The first annulus (1-2) must be green, since it's adjacent to the red disk. Then the next annulus (2-3) must be different from green, so red, then 3-4 green, etc. So indeed, the annuli alternate starting with green. Therefore, the coloring is:- Disk (1 cm): Red- Annulus 1-2: Green- Annulus 2-3: Red- Annulus 3-4: Green...So up to the 99-100 annulus. So total regions: 1 disk + 99 annuli = 100 regions. Wait, but the problem says "the circle with radius 1 cm is painted red, and all other areas bounded by consecutive circles are painted either red or green". So the disk is red, and the 99 annuli are alternately colored green and red. Therefore, starting with green for the first annulus (1-2). Then red (2-3), green (3-4), etc. So the colors alternate: green, red, green, red... for the annuli. Since they have to be different from the adjacent regions. The disk is red, so the first annulus (adjacent to the disk) must be green. Then the next annulus (adjacent to green) must be red, and so on. Therefore, the annuli alternate green and red. So the annuli from 1-2, 3-4, 5-6, etc., are green, and 2-3, 4-5, etc., are red. Wait, no: first annulus (1-2) is green, next (2-3) is red, (3-4) green, etc. So annulus k-(k+1) is green if k is odd, red if k is even? Wait, let's index the annuli. Let me think of the annulus between n and n+1 cm as the nth annulus. Then:- The first annulus (1-2 cm) is green.- The second annulus (2-3 cm) is red.- The third (3-4) green.- Etc.Therefore, annulus number n (between n and n+1 cm) is green if n is odd, red if n is even. Wait, but the first annulus (1-2) is n=1, green; second (2-3) n=2, red; yes. So the nth annulus is green if n is odd, red if n is even. However, the total number of annuli is 99, from 1-2 up to 99-100. Therefore, annulus numbers 1 to 99. So the coloring of the annuli alternates starting with green, red, green, red, etc. Therefore, the number of green annuli is the number of odd-numbered annuli, and red annuli are the even-numbered ones. Since there are 99 annuli, the number of green annuli is ceiling(99/2) = 50, and red annuli is 49? Wait, 99 divided by 2 is 49.5, so ceiling is 50. So 50 green annuli and 49 red annuli. But wait, annulus 1 is green, 2 is red,..., 99 is green (since 99 is odd). Therefore, yes, 50 green annuli and 49 red annuli.But also, we have the innermost disk, which is red. So total red regions: 1 (disk) + 49 (annuli) = 50. Total green regions: 50 (annuli). Therefore, the ratio of green area to red area would be (sum of green areas) / (sum of red areas). So need to calculate the total area of green regions and red regions.First, the red regions consist of the innermost disk (area π*(1)^2 = π) and the red annuli. Each red annulus is between even-numbered n and n+1, i.e., annulus 2-3, 4-5, ..., 98-99. Wait, no: annulus n is between n and n+1 cm. The red annuli are those with even n: n=2,4,...,98. Wait, n goes up to 99. Wait, annulus numbers 1 to 99. So red annuli are even numbers: 2,4,...,98. Wait, but 99 is odd. So the last red annulus would be 98-99? Wait, no. Wait, if n is the annulus number (1-99), then red annuli are those where n is even. So n=2,4,...,98. So 49 red annuli (since 98/2=49). Then annulus 99 is green. So the red annuli are the even-numbered ones from 2 to 98. So that's 49 annuli. Then green annuli are the odd-numbered ones from 1 to 99, which is 50 annuli. So red regions: 1 disk + 49 annuli = 50 regions. Green regions: 50 annuli. But the area of each annulus is different. The area of the disk is π*1²=π. The area of the annulus between k and k+1 cm is π*( (k+1)^2 - k^2 ) = π*(2k +1). So each annulus has area π*(2k +1), where k is the inner radius. Wait, for the annulus between k and k+1, inner radius is k, outer is k+1. So area is π*( (k+1)^2 - k^2 ) = π*(2k +1). Therefore, annulus k has area (2k +1)π. Wait, but annulus 1-2 has inner radius 1, so area (2*1 +1)π=3π. Annulus 2-3 is (2*2 +1)π=5π, etc. So each annulus between k and k+1 cm has area (2k +1)π.Therefore, to compute total red area: sum of the red annuli areas plus the innermost disk. The red annuli are the even-numbered ones (n=2,4,...,98). Each red annulus is between k and k+1 where k is the annulus number. Wait, no. Wait, if the annulus number is n (from 1 to 99), then the annulus between n and n+1 cm. So for annulus number n, the inner radius is n cm, outer radius is n+1 cm. Therefore, the area of annulus n is π*( (n+1)^2 - n^2 ) = π*(2n +1). So annulus n has area (2n +1)π. Therefore, if annulus n is red when n is even (since the first annulus, n=1, is green; then n=2 is red, n=3 green, etc.), so red annuli are annulus numbers 2,4,6,...,98. Each of those annuli has area (2n +1)π. Wait, but n here is the annulus number. Wait, annulus number 2 is between 2 and 3 cm, so inner radius 2 cm, outer radius 3 cm. So its area is π*(3² - 2²)=5π. But according to the formula, (2n +1)π where n=2: (4 +1)π=5π, which matches. So yes, annulus n has area (2n +1)π. Therefore, the red annuli are even n from 2 to 98, so n=2,4,...,98. How many terms? Since 98=2*49, there are 49 red annuli. Each red annulus n has area (2n +1)π. Therefore, the total red annuli area is π * sum_{k=1 to 49} (2*(2k) +1). Wait, let's substitute n=2k. Since n is even, let n=2k where k=1 to 49 (since n=2*1=2, up to n=2*49=98). Then area for each red annulus is (2*(2k) +1)π=(4k +1)π. Therefore, total red annuli area is π * sum_{k=1}^{49} (4k +1). Then the total red area is the disk (π) plus the red annuli areas: π + π * sum_{k=1}^{49} (4k +1).Similarly, green annuli are the odd-numbered annuli, n=1,3,5,...,99. There are 50 green annuli. Each green annulus n has area (2n +1)π. So total green area is π * sum_{k=1}^{50} (2*(2k -1) +1). Wait, n=2k -1 for k=1 to 50, since n is odd from 1 to 99. Then substituting, area is (2*(2k -1) +1)π=(4k -2 +1)π=(4k -1)π. Therefore, total green area is π * sum_{k=1}^{50} (4k -1).Therefore, the total red area is:Red area = π + π * sum_{k=1}^{49} (4k +1)Green area = π * sum_{k=1}^{50} (4k -1)So let's compute these sums.First, compute sum_{k=1}^{49} (4k +1). Let's break this into 4*sum(k) + sum(1):sum_{k=1}^{49} (4k +1) = 4*sum_{k=1}^{49}k + sum_{k=1}^{49}1Sum of k from 1 to n is n(n+1)/2. So here n=49:sum_{k=1}^{49}k = 49*50/2 = 1225sum_{k=1}^{49}1 = 49Therefore, sum_{k=1}^{49}(4k +1) = 4*1225 + 49 = 4900 + 49 = 4949Therefore, Red area = π + π*4949 = π*(1 + 4949) = π*4950Now Green area: sum_{k=1}^{50}(4k -1)Similarly, break into 4*sum(k) - sum(1):sum_{k=1}^{50}(4k -1) = 4*sum_{k=1}^{50}k - sum_{k=1}^{50}1Sum of k from 1 to 50 is 50*51/2 = 1275Sum of 1 fifty times is 50Therefore, sum = 4*1275 - 50 = 5100 - 50 = 5050Therefore, Green area = π*5050Therefore, the ratio of green area to red area is 5050π / 4950π = 5050/4950Simplify this fraction:Divide numerator and denominator by 50: 5050/50 = 101, 4950/50=99So 101/99. Wait, but 5050 divided by 50 is 101? 50*100=5000, so 5050-5000=50, which is 100 +1=101. Similarly, 4950 divided by 50 is 99. So yes, 101/99.But let me check the calculations again because this seems a bit strange. Let me verify the Green area sum:sum_{k=1}^{50}(4k -1) = 4*sum(k) - sum(1) = 4*(50*51/2) -50=4*(1275) -50=5100 -50=5050. Correct.Red area: sum_{k=1}^{49}(4k +1)=4*sum(k) +sum(1)=4*(49*50/2)+49=4*1225 +49=4900 +49=4949. Then total red area is 1 +4949=4950. Correct.Therefore, green is 5050, red is 4950. Ratio is 5050/4950=101/99≈1.0202...But let me think again. The total area of all regions is the area of the largest circle, which is π*100²=10000π. Let's check if red + green areas equal 10000π.Red area:4950π, Green area:5050π. 4950+5050=10000. Yes, correct. So the ratio is 5050/4950=101/99. So the answer is 101/99. So the ratio is 101:99, so 101/99.Wait, but let me think again: the red regions are the innermost disk (area π) plus 49 annuli. The green regions are 50 annuli. The annuli areas are increasing as the radius increases. So the green areas, being the later annuli, are larger. So the ratio should be slightly larger than 1, which 101/99≈1.02 is. So that makes sense.But just to be thorough, let me check with smaller numbers. Suppose there are only 2 circles: radii 1 and 2. Then the innermost disk (radius 1) is red. The annulus 1-2 must be green. Then the ratio would be green area (π*(2² -1²)=3π) over red area (π*1²=π). So 3π/π=3. But according to the formula, n=2 circles. Wait, wait in the original problem, it's 100 circles. Wait, maybe my example is different. Wait, in the original problem, if there are n circles, there are n-1 annuli. Wait, in my example, n=2 circles (radii 1 and 2), so 1 annulus (1-2). The innermost disk is red, the annulus is green. Then ratio green/red= 3π / π=3. But according to the formula, if n=2, then the red area would be 1 + sum_{k=1}^{(n-2)/2} ... but maybe the formula doesn't apply here. Wait, in the original problem with 100 circles, the formula gave us 101/99. Let's check with n=2. Wait, the number of annuli is 1 (since 2 circles). The red regions: disk (area π) and if any red annuli? Since the annulus 1-2 is green. So red area is π, green area is 3π. So ratio 3:1. If we use the formula from before, for n=2 circles, annuli=1. Then annulus 1 is green. So total green annuli=1, red annuli=0. Then red area=1π, green area= (4*1 -1)π=3π. Ratio 3π / π=3. So with n=2 circles, the ratio is 3. If we tried to apply the formula from the 100 circles case, 5050/4950=101/99≈1.02. But with n=2, the ratio is 3, which is different. Wait, but in the original problem, the formula was derived for 100 circles. So the formula's result depends on the number of circles. So for each n circles, the ratio would be different. So the answer for 100 circles is indeed 101/99. But let me check another example. Let's take n=3 circles (radii 1,2,3). Then regions: disk 1 (red), annulus 1-2 (green), annulus 2-3 (red). So red areas: disk (π) + annulus 2-3 (5π). Green area: annulus 1-2 (3π). So ratio green/red=3π / (π +5π)=3/6=1/2. But according to the formula: For n=3 circles, annuli=2. So green annuli=1 (n=1) and red annuli=1 (n=2). Red annuli area: annulus 2-3 (5π). Disk area=π. Total red=6π. Green annulus area=3π. Ratio=3/6=1/2. If we apply the formula:Red area: disk + sum_{k=1}^{(number of red annuli)}. Wait, with n=3 circles, the red annuli is annulus 2-3 (n=2). So in the formula, for n=3 circles, the number of annuli is 2. The red annuli are even-numbered annuli. But with n=3 circles, annuli are 1-2 (n=1) and 2-3 (n=2). So red annuli: n=2. Then sum_{k=1}^{1} (4k +1). Wait, because (n_annuli_red)=1. So 4*1 +1=5. Then red area=π +5π=6π. Green area: sum_{k=1}^{1} (4k -1). For green annuli n=1 (annulus 1-2). So 4*1 -1=3. Green area=3π. Ratio=3π/6π=1/2. So formula holds here. Similarly, if n=4 circles (radii 1,2,3,4). Then annuli: 1-2 (green), 2-3 (red), 3-4 (green). Red regions: disk (π) + annulus 2-3 (5π). Green regions: annulus 1-2 (3π) + annulus 3-4 (7π). So red total=6π, green total=10π. Ratio 10/6=5/3≈1.666...Using formula: For n=4 circles, annuli=3. Green annuli: n=1,3 (2 annuli). Red annuli: n=2 (1 annulus). So red area=π + sum_{k=1}^{1}(4k +1)=π + (4*1 +1)π=π +5π=6π. Green area=sum_{k=1}^{2}(4k -1)= (4*1 -1)+(4*2 -1)=3 +7=10. So 10π. Ratio 10/6=5/3. Correct. So formula works here. So for the original problem, 100 circles, gives 5050π green and 4950π red. Ratio 5050/4950=101/99≈1.0202. So answer is 101/99.Therefore, the ratio of green to red is 101:99, or 101/99.Final Answerboxed{dfrac{101}{99}}</think>Given 100 concentric circles with radii from 1 cm to 100 cm, the innermost circle (radius 1 cm) is painted red. The areas between consecutive circles (annuli) are painted either red or green such that adjacent areas have different colors. We need to determine the ratio of the total green area to the total red area.1. Coloring Pattern: - The innermost circle (radius 1 cm) is red. - The annuli alternate between green and red starting with green for the first annulus (1-2 cm).2. Regions and Areas: - The innermost circle (radius 1 cm) has an area of ( pi ). - Each annulus between radii ( k ) cm and ( k+1 ) cm has an area of ( pi((k+1)^2 - k^2) = pi(2k + 1) ).3. Number of Red and Green Annuli: - There are 99 annuli in total. - The innermost circle is red. - The annuli alternate colors: green, red, green, etc. - Green annuli: 50 (odd-numbered annuli: 1-2, 3-4, ..., 99-100). - Red annuli: 49 (even-numbered annuli: 2-3, 4-5, ..., 98-99).4. Calculating Areas: - Red Area: - Innermost circle: ( pi ). - Red annuli: Each red annulus ( k ) (even ( k )) has area ( pi(2k + 1) ). - Total red annuli area: ( sum_{k=1}^{49} pi(4k + 1) ). - Sum calculation: ( 4 sum_{k=1}^{49} k + sum_{k=1}^{49} 1 = 4 cdot frac{49 cdot 50}{2} + 49 = 4900 + 49 = 4949 pi ). - Total red area: ( pi + 4949 pi = 4950 pi ). - Green Area: - Green annuli: Each green annulus ( k ) (odd ( k )) has area ( pi(2k + 1) ). - Total green annuli area: ( sum_{k=1}^{50} pi(4k - 1) ). - Sum calculation: ( 4 sum_{k=1}^{50} k - sum_{k=1}^{50} 1 = 4 cdot frac{50 cdot 51}{2} - 50 = 5100 - 50 = 5050 pi ).5. Ratio: - The ratio of green area to red area is ( frac{5050 pi}{4950 pi} = frac{5050}{4950} = frac{101}{99} ).Thus, the ratio of the area of all green regions to the area of all red regions is (boxed{dfrac{101}{99}}).]

🔑:## Solution.The green areas are circular rings with a total area ofZ=2^{2} pi-1^{2} pi+4^{2} pi-3^{2} pi+6^{6} pi-5^{2} pi+ldots+100^{2} pi-99^{2} piThis can be written asbegin{aligned}Z & =pi((2-1)(2+1)+(4-3)(4+3)+(6-5)(6+5)+ldots+(100-99)(100+99))= & =pi((2+1)+(4+3)+(6+5)+ldots+(100+99))= & =pi(1+2+3+4+ldots+99+100)= & =pi(1+100) cdot 50=5050 piend{aligned}The area of the red regions isC=100^{2} pi-Z=10000 pi-5050 pi=4950 piThe desired ratio isfrac{Z}{C}=frac{5050 pi}{4950 pi}=frac{101}{99}

❓:Given the function f(x)=2aln x-x^{2}+1:1. If a=1, find the interval where the function f(x) is monotonically decreasing.2. If a gt 0, find the maximum value of the function f(x) on the interval left[1,+infty right).3. If f(x)leqslant 0 holds true on the interval left[1,+infty right), find the maximum value of a.

🧠:[<think>Okay, let's tackle these problems one by one. Starting with the first one: If ( a = 1 ), find the interval where the function ( f(x) = 2aln x - x^2 + 1 ) is monotonically decreasing. Alright, so when ( a = 1 ), the function becomes ( f(x) = 2ln x - x^2 + 1 ). To find where it's monotonically decreasing, I need to look at its derivative. If the derivative is negative, the function is decreasing there. Let's compute the derivative ( f'(x) ). The derivative of ( 2ln x ) is ( 2/x ), the derivative of ( -x^2 ) is ( -2x ), and the derivative of the constant 1 is 0. So, putting it all together:( f'(x) = frac{2}{x} - 2x )Now, we need to find where ( f'(x) < 0 ). Let's set up the inequality:( frac{2}{x} - 2x < 0 )Factor out 2:( 2left( frac{1}{x} - x right) < 0 )Divide both sides by 2 (which is positive, so inequality sign stays the same):( frac{1}{x} - x < 0 )Combine the terms:( frac{1 - x^2}{x} < 0 )So, the critical points are where numerator or denominator is zero: ( x = 0 ), ( x = 1 ), ( x = -1 ). But since the domain of ( f(x) ) is ( x > 0 ) (because of the ( ln x )), we can ignore ( x = 0 ) and ( x = -1 ). So critical point at ( x = 1 ).Now, we need to test the intervals around ( x = 1 ). Since the domain is ( x > 0 ), let's check intervals ( (0, 1) ) and ( (1, infty) ).For ( x in (0, 1) ), pick a test point, say ( x = 0.5 ):( frac{1 - (0.5)^2}{0.5} = frac{1 - 0.25}{0.5} = frac{0.75}{0.5} = 1.5 > 0 ), so the derivative is positive here.For ( x in (1, infty) ), pick ( x = 2 ):( frac{1 - 4}{2} = frac{-3}{2} = -1.5 < 0 ), so derivative is negative here.Therefore, ( f(x) ) is decreasing on ( (1, infty) ). However, the problem might want a closed interval if including the endpoint, but since it's monotonically decreasing, we usually state it as ( [1, infty) ), because at ( x = 1 ), the derivative is 0, so it's a critical point. But technically, monotonic decreasing can include points where the derivative is non-positive, so including x=1 is okay. So the interval is ( [1, infty) ). Wait, but let me confirm: the derivative at x=1 is ( 2/1 - 2*1 = 0 ). So right at x=1, the function has a horizontal tangent. But whether the interval is open or closed? Monotonically decreasing on an interval means that if you take any two points ( x_1 < x_2 ) in the interval, ( f(x_1) geq f(x_2) ). So even if at a point the derivative is zero, as long as the function doesn't increase after, it's still decreasing. So since to the right of 1, the derivative is negative, and at 1 the derivative is zero, the function is indeed decreasing on [1, ∞). So the answer is [1, ∞).Moving on to part 2: If ( a > 0 ), find the maximum value of the function ( f(x) ) on the interval [1, +∞).Alright, so now ( a ) is a positive constant, and we need to maximize ( f(x) = 2a ln x - x^2 + 1 ) on [1, ∞). To find the maximum, we can use calculus. First, find critical points by setting derivative to zero.Compute the derivative ( f'(x) = (2a)/x - 2x ). Set equal to zero:( (2a)/x - 2x = 0 )Multiply both sides by x (x > 0):( 2a - 2x^2 = 0 )Divide by 2:( a - x^2 = 0 implies x^2 = a implies x = sqrt{a} )So the critical point is at ( x = sqrt{a} ). Now, since we are looking at the interval [1, ∞), we need to check whether this critical point is in the interval. Since ( a > 0 ), ( sqrt{a} ) is positive. But if ( sqrt{a} geq 1 ), then the critical point is in the interval; else, if ( sqrt{a} < 1 ), which would mean ( a < 1 ), then the critical point is at x < 1, so not in our interval. Therefore, the maximum could occur either at x = sqrt(a) if sqrt(a) >=1, or at the endpoint x=1 otherwise.So let's split into cases:Case 1: ( a geq 1 ). Then critical point x = sqrt(a) is in [1, ∞). We need to check the value of f at x = sqrt(a) and compare with the endpoint x =1. Also, as x approaches infinity, what happens to f(x)? The term -x^2 dominates, so f(x) tends to -infinity. Hence, the maximum must be at either x=1 or x=sqrt(a).Case 2: 0 < a < 1. Then critical point x = sqrt(a) <1, so not in [1, ∞). So in this case, the maximum would occur either at x=1 or as x approaches infinity. But as x approaches infinity, f(x) tends to -infinity, so the maximum must be at x=1.Therefore, first, let's compute f at x=1:f(1) = 2a ln 1 - 1^2 +1 = 0 -1 +1 = 0.Next, if a >=1, compute f at x = sqrt(a):f(sqrt(a)) = 2a ln(sqrt(a)) - (sqrt(a))^2 +1Simplify:2a * (1/2) ln a - a +1 = a ln a -a +1.So, if a >=1, then the maximum is the larger of 0 and (a ln a -a +1). Wait, but we have to compare f(sqrt(a)) with f(1) =0. So when is a ln a -a +1 greater than 0?Let’s analyze the function g(a) = a ln a -a +1 for a >=1.Compute g(1) = 1*0 -1 +1 =0. Then compute derivative of g(a):g’(a) = ln a + a*(1/a) -1 = ln a +1 -1 = ln a.For a >=1, ln a >=0. Therefore, g’(a) >=0 for a >=1, which means g(a) is increasing on [1, ∞). Since g(1)=0, for a >1, g(a) >0. Therefore, for a >=1, f(sqrt(a)) = g(a) >=0, with equality only when a=1.So, in case a >=1, the maximum is at x=sqrt(a) and equals a ln a -a +1. However, when a=1, this value is 0, which is same as f(1). So for a >1, the maximum is at x=sqrt(a), and for a=1, both x=1 and x=sqrt(a)=1 give the same value. For 0 < a <1, the maximum is at x=1, which is 0.But the problem says "find the maximum value of the function f(x) on [1, ∞)". So depending on a:If a >=1, maximum is a ln a -a +1.If 0 < a <1, maximum is 0.But the problem states "if a >0, find the maximum value". So we need to write the maximum as a piecewise function?Wait, but maybe they want the overall maximum, but perhaps not. The problem says "the maximum value of the function f(x) on the interval [1, +∞)". So it's a function of a. So the answer is:Maximum value is:0, if 0 < a <=1,a ln a -a +1, if a >=1.But maybe check whether for a >=1, the value a ln a -a +1 is indeed positive?Wait, earlier when analyzing, for a >1, since g(a) = a ln a -a +1 is increasing and g(1)=0, then yes, it's positive for a >1. So that's the case.But hold on, when a approaches infinity, what happens to the maximum value? Let's see: a ln a -a +1. The dominant term is a ln a -a. As a approaches infinity, this tends to infinity*(infinity -1) which would be infinity. Wait, but is that true?Wait, a ln a -a = a (ln a -1). As a increases, ln a increases, so (ln a -1) becomes positive once a > e, and then grows to infinity. So yes, a ln a -a +1 tends to infinity as a approaches infinity. Therefore, the maximum value goes to infinity as a increases.But the question is asking for the maximum value of f(x) on [1, ∞) given that a >0. So depending on a, the maximum is either 0 or a ln a -a +1. So the answer is the maximum is max{0, a ln a -a +1}.But perhaps the problem expects a single expression? Wait, maybe not. Let me check again.Wait, perhaps the question is expecting a specific value? Wait, no. For each a >0, find the maximum value. So the maximum value is 0 when a <=1, and a ln a -a +1 when a >=1. So the maximum value is:( begin{cases} 0 & text{if } 0 < a leq 1, a ln a - a +1 & text{if } a geq 1. end{cases} )But maybe express it in terms of a maximum function? Hmm.Alternatively, since when a >1, the maximum is at x=sqrt(a), and since the function tends to -infty as x increases, so that's the global maximum on [1, ∞). So the answer is max{0, a ln a -a +1}. But I think the problem expects an answer in boxed notation. However, maybe it's not piecewise. Wait, let me check if when a=1, both expressions give 0, so that's consistent.Alternatively, maybe for all a>0, the maximum value is a ln a -a +1, but when a <=1, this expression is less than or equal to 0. Let me check:Take a=1: 1*0 -1 +1 =0.Take a=0.5: 0.5 ln 0.5 -0.5 +1. ln 0.5 is negative, so 0.5*(-0.6931) -0.5 +1 ≈ -0.3466 -0.5 +1 ≈ 0.1534. Wait, but that contradicts previous analysis. Wait, hold on, if a=0.5, which is less than 1, according to our previous analysis, the maximum should be at x=1, which is 0. But according to this expression a ln a -a +1 when a=0.5 is approx 0.1534, which is positive, but according to the function, since critical point x=sqrt(0.5) ≈0.707 <1, so not in the interval [1, ∞). Therefore, the maximum on [1, ∞) is at x=1, which is 0. Therefore, the expression a ln a -a +1 for a <1 is positive, but the actual maximum on [1, ∞) is 0. Therefore, the correct maximum is the maximum between 0 and a ln a -a +1, but only when the critical point is in the interval. Wait, perhaps another way: when the critical point is in the interval [1, ∞), which is when sqrt(a) >=1, i.e., a >=1, then the maximum is a ln a -a +1. Otherwise, when a <1, the maximum is at x=1, which is 0. Therefore, the answer is:If a >=1, the maximum is a ln a -a +1; if a <1, the maximum is 0. So in boxed form, maybe present both cases? But the problem says "find the maximum value", so likely the answer is expressed as a piecewise function. However, the problem might expect just the expression for a>0, considering that when a>1, the maximum is at x=sqrt(a), and when 0 <a <=1, the maximum is at x=1. So to write the maximum value, it's:( max left{ 0, a ln a - a + 1 right} )But if the problem expects a specific value depending on a, then maybe the answer is written as:- For ( a geq 1 ), the maximum value is ( a ln a - a + 1 )- For ( 0 < a < 1 ), the maximum value is ( 0 )But since part 2 says "if ( a > 0 ), find the maximum value", perhaps the answer is the maximum of these two, but since for a >1, the value is larger, but depending on a. So perhaps the answer is written as:( begin{cases} 0 & text{if } 0 < a leq 1, a ln a - a + 1 & text{if } a > 1. end{cases} )But maybe the problem expects the expression a ln a -a +1, but only when it's positive. Alternatively, note that when a <=1, the maximum is 0, and when a >1, it's a ln a -a +1. So, the answer is boxed{a ln a - a + 1} if a >=1, and boxed{0} if a <=1. But the problem says "if a >0, find the maximum value". Since the answer depends on a, we need to write it as a piecewise function. However, the problem may have intended to just consider the case where the critical point is in the interval, but maybe not. Wait, let me check if for a <1, even though the critical point is not in [1, ∞), the maximum is at x=1. Therefore, the answer is 0 for a <=1 and a ln a -a +1 for a >=1. So the maximum value is:( boxed{0} ) when ( 0 < a leq 1 ), and ( boxed{a ln a - a + 1} ) when ( a geq 1 ).But the problem says "find the maximum value", so maybe they expect the answer expressed in terms of a, which would be the piecewise function. However, the way the problem is phrased, it's possible that they expect the answer as a function of a, so the answer is either 0 or the other expression. Since we are to put the final answer in a box, and the problem is split into three parts, part 2 is:2. If ( a >0 ), find the maximum value of the function ( f(x) ) on the interval ( [1, +infty) ).So, since the answer depends on a, and the problem allows for a general a>0, perhaps they expect the answer as a piecewise function. But since LaTeX doesn't handle piecewise functions in a single box easily, maybe the problem expects the expression in terms of a, but considering that when a <=1, the maximum is 0, and when a >=1, it's a ln a -a +1. However, the problem may not want a piecewise function but instead the maximum possible value. Wait, but the maximum value varies with a. Wait, no, given a, find the maximum of f(x). So, for each a>0, compute the maximum. Therefore, the answer is:If ( a geq 1 ), the maximum value is ( a ln a - a + 1 ); if ( 0 < a <1 ), the maximum value is 0.But how to write this in a boxed format? Maybe two separate boxed answers? But the original instruction says "put your final answer within boxed{}". So perhaps the answer is written as a piecewise function:boxed{ begin{cases} 0 & text{if } 0 < a leq 1, a ln a - a + 1 & text{if } a geq 1. end{cases} }But sometimes, in some contexts, they might expect just the expression, acknowledging that it's conditional. Alternatively, if the problem expects only the expression where the maximum is achieved at the critical point, but considering that for a <1, the maximum is 0, which is less than the value at critical point (but critical point is outside the interval). Hmm. Wait, actually, if we consider that for a <1, the critical point is at x=sqrt(a) <1, but in the interval [1, ∞), the maximum is at x=1. For a >=1, the critical point is within the interval, so maximum there. So yes, the answer is as above. Therefore, in boxed notation, as per instruction, since the problem is split into three parts, part 2 should be answered with the boxed expression. However, given that the answer is piecewise, but standard practice here might be to accept piecewise in a box. Alternatively, maybe there's a different approach. Wait, but maybe the problem wants the maximum value in terms of a without conditions, but that's not possible. Since it's conditional on a. So the correct answer is a piecewise function.Alternatively, perhaps we can write it using the maximum function: boxed{maxleft{0, a ln a - a + 1right}}. But depending on interpretation. In many calculus problems, when you have endpoints and critical points, you evaluate the function at all critical points in the interval and endpoints, then pick the largest. So here, the maximum value is the maximum between f(1)=0 and f(sqrt(a)) when sqrt(a)>=1. Therefore, the answer is boxed{maxleft{0, a ln a - a + 1right}}.But in Chinese textbooks or some contexts, they might prefer the piecewise. However, given that the problem is in English and given the way it's phrased, maybe expressing it as the maximum of 0 and a ln a -a +1 is acceptable. Let's check with a=2: f(sqrt(2))=2 ln 2 -2 +1≈ 1.386 -1=0.386, which is positive, and f(1)=0, so maximum is 0.386. If a=0.5: maximum is 0, since critical point is at x=sqrt(0.5)≈0.707 <1, so f(1)=0, and f(x) decreases afterwards. So yes, the maximum is max{0, a ln a -a +1}. Therefore, the boxed answer is boxed{maxleft{0, a ln a - a + 1right}}.But let me confirm if "maximum value" would be written that way. Alternatively, perhaps in the problem's context, they consider that for a >0, the maximum is achieved at x=1 if a <=1, and at x=sqrt(a) otherwise, so writing the piecewise function is better. However, since the user might expect a single boxed answer, and given the limitations of boxing piecewise functions, maybe the answer is boxed{a ln a - a + 1} with an understanding that when a <=1, the maximum is 0. But since part 3 is about f(x) <=0 on [1, ∞), which would require that the maximum is <=0, which relates to a being such that a ln a -a +1 <=0. So maybe in part 2, the answer is a ln a -a +1, but with the caveat that if a <=1, it's 0, but in boxed notation... Hmm. Maybe the problem expects the answer for part 2 as boxed{a ln a - a + 1}, since that's the maximum when a >=1, and when a <1, it's 0, but since the problem says "if a >0", maybe just writing the general expression, knowing that when a <1, the critical point is not in the interval. However, the function a ln a -a +1 when a <1 is actually positive (since as we saw for a=0.5, it's approximately 0.15), but the actual maximum on [1, ∞) is 0, so that expression is not valid for a <1. Therefore, the correct answer is indeed a piecewise function, but since we need to put the final answer in a single box, maybe the problem expects just the expression for the maximum when the critical point is in the interval, but that's not correct. Therefore, the safest way is to write the piecewise function boxed. But how?Alternatively, since in part 3, they refer to the maximum value of a such that f(x) <=0 on [1, ∞), which would be when the maximum of f(x) is 0, which occurs when a ln a -a +1 <=0, which would be for a <=1. But maybe that's part 3.Given the confusion, perhaps the answer to part 2 is boxed{a ln a - a + 1} if a >=1 and boxed{0} otherwise. But given that part 2 is a separate question, and given the original problem structure, maybe it's acceptable to write the piecewise function inside a single box using cases. In LaTeX, that's possible with the cases environment. So:The maximum value is (boxed{begin{cases} 0 & text{if } 0 < a leq 1, a ln a - a + 1 & text{if } a geq 1. end{cases}})But check if the system supports that. Since the user mentioned to put the final answer within boxed{}, perhaps using a single box with cases. Alternatively, if the system doesn't accept cases inside a box, then the answer would have to be split. But given that in some online platforms, you can use boxed{} with cases, I think that's acceptable. So, proceeding with that.Now, part 3: If ( f(x) leq 0 ) holds on the interval ( [1, +infty) ), find the maximum value of ( a ).So we need to find the largest a such that for all x >=1, f(x) <=0. From part 2, we know that when a <=1, the maximum of f(x) on [1, ∞) is 0, achieved at x=1. For a >1, the maximum is a ln a -a +1. Therefore, to have f(x) <=0 for all x >=1, we need that the maximum value of f(x) on [1, ∞) is <=0. Therefore, when a <=1, maximum is 0 (so f(x) <=0 holds), but when a >1, the maximum is a ln a -a +1, which must be <=0. So we need to solve:a ln a -a +1 <=0, for a >1.So find the maximum a >1 such that a ln a -a +1 <=0.Let’s denote h(a) = a ln a -a +1. We need to find the largest a where h(a) =0.Compute h(1) = 0. We saw earlier that h(a) increases for a >=1 (since h’(a)=ln a), so h(a) is increasing on [1, ∞). Wait, h’(a) = derivative of a ln a -a +1 = ln a +1 -1 = ln a. So for a >1, h’(a) >0. Therefore, h(a) is increasing on [1, ∞). At a=1, h(a)=0. For a >1, h(a) increases to infinity. Therefore, the equation h(a)=0 has only solution at a=1. Therefore, for a >1, h(a) >0, which violates the condition f(x) <=0. But wait, this contradicts the previous analysis. Wait, if h(a) = a ln a -a +1 is increasing for a >=1, starting at h(1)=0 and going to infinity, then for a >1, h(a) >0, which would mean that f(x) achieves positive values, hence violating f(x) <=0. Therefore, the only value where f(x) <=0 on [1, ∞) is when a <=1. Therefore, the maximum value of a is 1.But wait, when a=1, the maximum value is 0, which satisfies f(x) <=0. If a is slightly larger than 1, say a=1.1, then h(a)=1.1 ln1.1 -1.1 +1 ≈1.1*0.0953 -0.1≈0.1048 -0.1≈0.0048>0, so f(x) would exceed 0. Therefore, the maximum a is 1.But let me verify with a=1. At a=1, f(x)=2 ln x -x^2 +1. We need to check if f(x) <=0 for all x >=1.Compute f(1)=0. For x >1, compute f(x). Take x approaching 1 from the right: the derivative at x=1 is 0, and for x >1, the function is decreasing (from part 1). Therefore, f(x) decreases from 0 to -infinity as x approaches infinity. Hence, for a=1, f(x) <=0 for all x >=1. Therefore, a=1 is acceptable.If a >1, then h(a)=a ln a -a +1 >0, so f(x) reaches a positive value at x=sqrt(a), which violates f(x) <=0. If a <1, then the maximum of f(x) on [1, ∞) is 0 (attained at x=1), but wait, if a <1, then f(1)=0, and since for x >1, the derivative is f’(x)=2a/x -2x. At x=1, f’(1)=2a -2. If a <1, then 2a -2 <0, so the function is decreasing at x=1. Therefore, for a <1, f(x) starts at 0 when x=1 and decreases afterwards. Therefore, f(x) <=0 for all x >=1. Hence, when a <=1, f(x) <=0 on [1, ∞). Therefore, the maximum value of a is 1.But wait, the question says "if f(x) <=0 on [1, ∞), find the maximum value of a". So the answer is 1. But let me verify with a=1, it's the threshold. So yes, the maximum a is 1.Thus, the answers are:1. [1, ∞)2. Piecewise function: 0 for 0 <a <=1 and a ln a -a +1 for a >=13. 1But let me check for part 3 again. Suppose a=1, then f(x)=2 ln x -x^2 +1. We need to ensure that f(x) <=0 for all x >=1. Let's check x=1: f(1)=0. For x>1, as established in part1, the function is decreasing, so f(x) <0 for x>1. Therefore, yes, when a=1, f(x) <=0 on [1, ∞). If a>1, then the maximum of f(x) is positive, so it violates the condition. If a<1, the maximum is 0, but when a<1, f(x) is decreasing from 0, so f(x) <=0 for all x >=1. Wait, but if a<1, then f(1)=0, but is f(x) <=0 for all x >=1?Wait, if a <1, then at x=1, f(1)=0, and derivative at x=1 is f’(1)=2a -2 <0 (since a<1), so the function is decreasing at x=1. Therefore, for x>1, f(x) decreases from 0, hence f(x) <0 for x>1. Therefore, for a <=1, f(x) <=0 on [1, ∞). Hence, the maximum a is 1.Therefore, part 3 answer is boxed{1}.Final Answer1. boxed{[1, +infty)}2. boxed{begin{cases} 0 & text{if } 0 < a leq 1, a ln a - a + 1 & text{if } a geq 1 end{cases}}3. boxed{1}</think>Given the function ( f(x) = 2a ln x - x^2 + 1 ):1. If ( a = 1 ), find the interval where the function ( f(x) ) is monotonically decreasing. When ( a = 1 ), the function becomes ( f(x) = 2 ln x - x^2 + 1 ). To determine where it is monotonically decreasing, we compute the derivative: [ f'(x) = frac{2}{x} - 2x ] Setting ( f'(x) < 0 ): [ frac{2}{x} - 2x < 0 implies frac{1 - x^2}{x} < 0 ] The critical points are ( x = 1 ). Testing intervals around ( x = 1 ), we find that ( f'(x) < 0 ) for ( x > 1 ). Since ( f(x) ) is decreasing at ( x = 1 ) and continues to decrease for ( x > 1 ), the interval is: [ boxed{[1, +infty)} ]2. If ( a > 0 ), find the maximum value of the function ( f(x) ) on the interval ([1, +infty)). The derivative is: [ f'(x) = frac{2a}{x} - 2x ] Setting ( f'(x) = 0 ): [ frac{2a}{x} - 2x = 0 implies x = sqrt{a} ] For ( a geq 1 ), the critical point ( x = sqrt{a} ) is within the interval ([1, +infty)). Evaluating ( f(x) ) at ( x = sqrt{a} ): [ f(sqrt{a}) = a ln a - a + 1 ] For ( 0 < a < 1 ), the critical point is outside the interval, and the maximum value is at ( x = 1 ): [ f(1) = 0 ] Thus, the maximum value is: [ boxed{begin{cases} 0 & text{if } 0 < a leq 1, a ln a - a + 1 & text{if } a geq 1 end{cases}} ]3. If ( f(x) leq 0 ) holds true on the interval ([1, +infty)), find the maximum value of ( a ). For ( f(x) leq 0 ) on ([1, +infty)), the maximum value of ( f(x) ) must be 0. From part 2, the maximum value for ( a geq 1 ) is ( a ln a - a + 1 ). Setting this to be non-positive: [ a ln a - a + 1 leq 0 ] This holds true only at ( a = 1 ). For ( a > 1 ), the maximum value is positive, violating the condition. Thus, the maximum value of ( a ) is: [ boxed{1} ],<think>Okay, let's tackle these three problems one by one. They all relate to the function ( f(x) = 2aln x - x^2 + 1 ). Let's start with the first one.1. If ( a = 1 ), find the interval where the function ( f(x) ) is monotonically decreasing.Alright, so when ( a = 1 ), the function becomes ( f(x) = 2ln x - x^2 + 1 ). To find where it's monotonically decreasing, I need to look at its derivative. If the derivative is negative on an interval, the function is decreasing there.Let's compute the derivative ( f'(x) ). The derivative of ( 2ln x ) is ( 2/x ), the derivative of ( -x^2 ) is ( -2x ), and the derivative of the constant 1 is 0. So,( f'(x) = frac{2}{x} - 2x ).Now, we need to find where ( f'(x) leq 0 ).Set ( frac{2}{x} - 2x leq 0 ).Multiply both sides by ( x ) (since ( x > 0 ) because the domain of ( ln x ) is ( x > 0 )), which gives:( 2 - 2x^2 leq 0 )Simplify:( 2 leq 2x^2 )Divide both sides by 2:( 1 leq x^2 )So,( x^2 geq 1 )Taking square roots,( |x| geq 1 ).But since ( x > 0 ), this simplifies to ( x geq 1 ).Therefore, the function ( f(x) ) is monotonically decreasing on ( [1, +infty) ).Wait, let me check that again. When we multiplied both sides by ( x ), which is positive, so the inequality direction remains the same. So yes, ( 2 - 2x^2 leq 0 ) leads to ( x^2 geq 1 ), so ( x geq 1 ).Hence, the interval is ( [1, infty) ). That seems straightforward.2. If ( a > 0 ), find the maximum value of the function ( f(x) ) on the interval ( [1, +infty) ).Hmm. To find the maximum value, we need to look for critical points in the interval and evaluate the function at those points as well as at the endpoints.First, let's find the derivative of ( f(x) ) for general ( a ):( f'(x) = frac{2a}{x} - 2x ).Set the derivative equal to zero to find critical points:( frac{2a}{x} - 2x = 0 ).Multiply both sides by ( x ):( 2a - 2x^2 = 0 )Divide by 2:( a - x^2 = 0 )So,( x^2 = a )Thus,( x = sqrt{a} ).Since ( x > 0 ), we only consider the positive root.Now, we need to check if ( sqrt{a} ) is in the interval ( [1, +infty) ). That depends on the value of ( a ). Since ( a > 0 ), ( sqrt{a} geq 1 ) if and only if ( a geq 1 ). If ( 0 < a < 1 ), then ( sqrt{a} < 1 ), so the critical point would be outside the interval ( [1, +infty) ).Therefore, there are two cases:Case 1: ( a geq 1 )The critical point ( x = sqrt{a} ) is in the interval ( [1, +infty) ). Therefore, the maximum could be at ( x = sqrt{a} ) or at the endpoint ( x = 1 ). However, since we are looking for a maximum on ( [1, +infty) ), and as ( x to +infty ), ( f(x) ) behaves like ( -x^2 ) (since ( 2aln x ) grows much slower than ( -x^2 )), so ( f(x) to -infty ). Therefore, the maximum must occur either at the critical point ( x = sqrt{a} ) or at ( x = 1 ).Case 2: ( 0 < a < 1 )Then the critical point ( x = sqrt{a} < 1 ), which is not in the interval. Therefore, the function is decreasing on the entire interval ( [1, +infty) ) because the derivative ( f'(x) = frac{2a}{x} - 2x ). At ( x = 1 ), ( f'(1) = 2a - 2 ). Since ( a < 1 ), this is negative. And since the derivative is decreasing (as we can check second derivative or note that as ( x ) increases, ( f'(x) ) becomes more negative). Therefore, the function is decreasing on ( [1, infty) ), so the maximum is at ( x = 1 ).Therefore, summarizing:- If ( a geq 1 ), check ( x = sqrt{a} ) and ( x = 1 ).- If ( 0 < a < 1 ), maximum at ( x = 1 ).But the problem says ( a > 0 ), so we need to handle both cases.Wait, but maybe even for ( a geq 1 ), we need to check whether the critical point is a maximum. Let's confirm by checking the second derivative or the sign changes.Compute the second derivative:( f''(x) = -frac{2a}{x^2} - 2 ).This is always negative because ( a > 0 ), so ( -frac{2a}{x^2} ) is negative and ( -2 ) is negative. Hence, ( f''(x) < 0 ) for all ( x > 0 ). Therefore, any critical point is a local maximum.Therefore, for ( a geq 1 ), the critical point ( x = sqrt{a} ) is a local maximum, and since the function tends to ( -infty ) as ( x to infty ), this must be the global maximum on ( [1, infty) ). For ( 0 < a < 1 ), there's no critical point in the interval, so the maximum is at ( x = 1 ).Therefore, the maximum value is:- ( f(sqrt{a}) ) if ( a geq 1 )- ( f(1) ) if ( 0 < a < 1 )Let's compute both.First, ( f(1) = 2a ln 1 - 1^2 + 1 = 0 - 1 + 1 = 0 ).Wait, ( ln 1 = 0 ), so yeah, ( f(1) = 0 ).Now, for ( a geq 1 ), compute ( f(sqrt{a}) ):( f(sqrt{a}) = 2a ln sqrt{a} - (sqrt{a})^2 + 1 )Simplify:( 2a cdot frac{1}{2} ln a - a + 1 = a ln a - a + 1 )So, the maximum value is ( a ln a - a + 1 ) when ( a geq 1 ), and 0 when ( 0 < a < 1 ).But the problem says "find the maximum value of the function ( f(x) ) on the interval ( [1, +infty) )" given ( a > 0 ). So depending on ( a ), we need to give the appropriate expression.But the problem might be expecting a single expression, but likely we need to express it piecewise or determine the overall maximum.Wait, but maybe the maximum is the larger of ( 0 ) and ( a ln a - a + 1 ). But actually, when ( a geq 1 ), is ( a ln a - a + 1 ) greater than 0?Let's check at ( a = 1 ):( 1 ln 1 -1 +1 = 0 -1 +1 = 0 ), which matches ( f(1) ). For ( a >1 ), let's take ( a = e ):( e cdot 1 - e +1 = e - e +1 =1 ), which is positive. So when ( a >1 ), the maximum is positive, and when ( a =1 ), it's zero. For ( a <1 ), maximum is 0. Therefore, the maximum value is:( begin{cases} a ln a -a +1 & text{if } a geq1 0 & text{if } 0 <a <1 end{cases} )But maybe we can write it more concisely. Let's see. Since for ( a geq1 ), ( a ln a -a +1 ) is the maximum, and for ( 0 <a <1 ), the maximum is 0.Alternatively, we can note that ( a ln a -a +1 ) is equal to (1 - a + a ln a). For ( a geq1 ), this is the maximum, and for ( 0 <a <1 ), it's 0.But maybe the problem is expecting the answer in terms of ( a ). So depending on ( a ), the maximum is either 0 or ( a ln a -a +1 ). Since the problem says "find the maximum value", perhaps we need to write both cases. But let's check if there's a mistake here.Wait, if ( a geq1 ), the maximum is at ( x = sqrt{a} ), which gives ( a ln a -a +1 ). For ( 0 <a <1 ), the maximum is at ( x =1 ), which is 0. So the answer is a piecewise function. However, the problem doesn't specify whether ( a ) is a given constant or if they want the maximum in terms of ( a ). Since part 3 is about finding the maximum ( a ) such that ( f(x) leq 0 ), maybe here in part 2, it's just expressing the maximum value in terms of ( a ), so the answer is:The maximum value is ( max{0, a ln a - a +1} ). But perhaps they want the expression in terms of ( a ), so the answer is:If ( a geq1 ), the maximum is ( a ln a -a +1 ); if ( 0 <a <1 ), the maximum is 0.But let's check with ( a =2 ):( f(x)=4 ln x -x^2 +1 ). The critical point is at ( x = sqrt{2} ). Compute ( f(sqrt{2}) =4 ln sqrt{2} - (sqrt{2})^2 +1 =4*(0.5 ln 2) -2 +1 =2 ln 2 -1 ≈2*0.693 -1≈1.386 -1≈0.386 ). Which is positive. So the maximum is 0.386 for ( a=2 ).For ( a=0.5 ), the maximum is at ( x=1 ), which is 0. So yes, the answer is piecewise.But maybe the problem expects the answer to be written as the maximum of those two. Alternatively, since the problem says "find the maximum value", perhaps we can write it as:The maximum value is ( begin{cases} a ln a - a +1, & a geq1 0, & 0 <a <1 end{cases} ).But in the problem statement, part 3 is to find the maximum ( a ) such that ( f(x) leq0 ) on ( [1, +infty) ). So part 2 is probably a stepping stone for part 3. So maybe in part 2, the answer is that the maximum value is ( a ln a -a +1 ) when ( a geq1 ), and 0 otherwise, but since the problem says "if ( a >0 ), find the maximum value", perhaps they want to write the maximum as:( max {0, a ln a - a +1 } ).But in mathematical terms, it's piecewise. Since the problem is in Chinese (translated), maybe they expect the answer as:When ( a geq1 ), the maximum is ( a ln a -a +1 ); when ( 0 <a <1 ), the maximum is 0.But I need to check if that's correct. Let me verify with a test value.Take ( a=1 ): maximum is (1*0 -1 +1=0). Correct, same as ( x=1 ).Take ( a=2 ): as above, approximately 0.386, which is positive. So when ( a geq1 ), the maximum is positive. When ( a <1 ), it's 0.Therefore, the maximum value is ( max(0, a ln a -a +1) ). But perhaps the problem expects the answer as ( a ln a -a +1 ) when ( a geq1 ), else 0. However, in mathematical writing, we often use piecewise functions. So the answer should be presented as a piecewise function.But let me check the exact question wording: "find the maximum value of the function ( f(x) ) on the interval ( [1, +infty) )".So depending on the value of ( a ), the maximum is different. Therefore, the answer is:The maximum value is ( a ln a -a +1 ) if ( a geq1 ), and 0 if ( 0 <a <1 ).Alternatively, since when ( a geq1 ), the expression ( a ln a -a +1 ) is positive, and when ( a <1 ), the maximum is 0, which is less than the other case. Therefore, the maximum value is ( max(0, a ln a -a +1) ). However, in exams, often they prefer piecewise functions. So maybe the answer is written as:( begin{cases} a ln a - a + 1 & text{if } a geq1, 0 & text{if } 0 < a <1. end{cases} )But the problem didn't specify to write it as a piecewise function; it just said "find the maximum value". So perhaps we need to write both possibilities. Alternatively, since when ( a geq1 ), the maximum is ( a ln a -a +1 ), and when ( a <1 ), the maximum is 0. Therefore, the answer is either of those expressions depending on ( a ). But the problem says "if ( a >0 )", so ( a ) is given, and we have to express the maximum in terms of ( a ). Therefore, the answer is a piecewise function. Since the user might expect the answer written in boxed notation, but boxed notation for piecewise is difficult. Maybe they just expect the expression for when it's non-zero? Wait, let me check the problem again.Wait, the second problem says: "If ( a >0 ), find the maximum value of the function ( f(x) ) on the interval ( [1, +infty) )." So perhaps it's expecting an answer in terms of ( a ), considering both cases.But maybe in Chinese math problems, sometimes they expect you to recognize that the maximum is at ( x = sqrt{a} ) if ( sqrt{a} geq1 ), else at ( x=1 ), and hence the maximum value is ( max{0, a ln a -a +1} ). But since ( a ln a -a +1 ) is negative when ( a <1 ), but wait, no.Wait, wait. Let's evaluate ( a ln a -a +1 ) when ( a <1 ). Let's take ( a = 0.5 ):( 0.5 ln 0.5 -0.5 +1 approx0.5*(-0.693) -0.5 +1 ≈-0.3465 -0.5 +1 ≈0.1535 ). Wait, that's positive. Wait, but when ( a <1 ), we said the maximum is at ( x=1 ), which is 0. So there's a discrepancy here.Wait, this suggests that my previous conclusion might be wrong. Let's check.If ( a =0.5 ), then the critical point is ( x = sqrt{0.5} ≈0.707 ), which is less than 1, so not in the interval. Therefore, on the interval [1, ∞), the function is decreasing because the derivative at x=1 is ( 2a -2*1 = 2a -2 ). For a=0.5, this is 1 -2 = -1 <0. Thus, the function is decreasing on [1, ∞), so the maximum is at x=1, which is f(1)=0. However, if we compute ( a ln a -a +1 ) for a=0.5, we get approximately 0.1535, which is positive, but that occurs at x=√0.5, which is not in our interval. Therefore, in the interval [1, ∞), the maximum is indeed 0 for a <1, and a ln a -a +1 for a ≥1. But when a <1, the value at the critical point is positive, but since the critical point is outside the interval, the maximum on [1, ∞) is still 0. Therefore, the conclusion is correct.Therefore, the maximum value is:- For a ≥1: a ln a -a +1- For 0 <a <1: 0But let's check with a=0.5. If we compute f(1)=0. If we compute f(x) at x=2:f(2)=2*0.5 ln2 -4 +1= ln2 -3 ≈0.693 -3≈-2.307, which is less than 0. So yes, the maximum is 0 at x=1.Therefore, the answer is correct.3. If ( f(x)leq 0 ) holds true on the interval ( [1, +infty) ), find the maximum value of ( a ).Okay, so we need to find the largest ( a ) such that ( f(x) leq0 ) for all ( x geq1 ).From part 2, we know that the maximum of ( f(x) ) on [1, ∞) is:- ( a ln a -a +1 ) if ( a geq1 )- 0 if ( 0 <a <1 )But wait, if ( a <1 ), the maximum is 0, so ( f(x) leq0 ) would not hold because the maximum is 0, which is equal to 0. But the problem says ( f(x)leq0 ). So if the maximum is 0, then ( f(x) leq0 ) holds. Wait, but when a <1, the maximum is 0? Wait, let's confirm.Wait, no, earlier conclusion: For ( 0 <a <1 ), the maximum of ( f(x) ) on [1, ∞) is 0. But when a <1, the critical point is at x=√a <1, so on [1, ∞), the function is decreasing (since derivative is negative at x=1 and becomes more negative). Therefore, f(x) is decreasing on [1, ∞), starting from f(1)=0 and going to -∞. Therefore, the maximum on [1, ∞) is 0. Therefore, if a <1, then f(x) ≤0 on [1, ∞) because the maximum is 0. However, if a ≥1, the maximum is a ln a -a +1. So for f(x) ≤0, we need:For a ≥1: a ln a -a +1 ≤0For 0 <a <1: automatically holds since maximum is 0.Therefore, the maximum value of a is the largest a such that a ln a -a +1 ≤0. Since when a=1, a ln a -a +1=0 -1 +1=0. For a>1, we need to check when a ln a -a +1 ≤0.Therefore, solve a ln a -a +1 ≤0 for a ≥1.Let’s define the function g(a) = a ln a -a +1 for a ≥1.We need to find the maximum a such that g(a) ≤0.Check g(1)=0.Check the derivative of g(a):g’(a) = ln a + a*(1/a) -1 +0 = ln a +1 -1 = ln a.Thus, g’(a) = ln a.For a >1, ln a >0, so g(a) is increasing for a >1.Therefore, since g(1)=0 and g(a) increases for a >1, g(a) >0 for a >1. Therefore, the only a where g(a) ≤0 is a=1. Therefore, the maximum value of a is 1.But wait, hold on. Wait, but when a=1, the maximum of f(x) is 0, which satisfies f(x) ≤0. For any a >1, the maximum of f(x) is positive, which violates f(x) ≤0. For a <1, the maximum is 0, but f(x) ≤0 on [1, ∞). Wait, but when a <1, even though the maximum is 0, the function is decreasing from 0 to -∞, so f(x) ≤0. However, for a=1, the maximum is 0, but the function f(x) at a=1 is 2 ln x -x² +1. Let's check its behavior.Wait, at a=1, the critical point is at x=1. The second derivative is negative, so it's a maximum. Wait, when a=1, the critical point x= sqrt(a)=1. So f(x) has a critical point at x=1. Since the second derivative is negative, it's a local maximum. So f(1)=0, and then it decreases to -infty. Therefore, at a=1, f(x) ≤0 on [1, ∞). If a is slightly less than 1, say a=0.9, then the critical point is at sqrt(0.9)≈0.948, which is less than 1. Then on [1, ∞), the function is decreasing, starting at f(1)=0 and going to -infty. Therefore, for a ≤1, f(x) ≤0 on [1, ∞). For a>1, the function has a maximum at x=sqrt(a)>1, which is positive, hence violating f(x) ≤0. Therefore, the maximum value of a is 1.But wait, the problem says "if f(x) ≤0 holds on [1, ∞), find the maximum value of a". Therefore, the answer is a=1.But let me verify with a=1. When a=1, f(x)=2 ln x -x² +1. Let's check if f(x) ≤0 for all x ≥1.At x=1: f(1)=0-1+1=0.For x>1: The function is decreasing (from part 1, when a=1, the function is monotonically decreasing on [1, ∞)), so f(x) ≤0 for x ≥1.Therefore, yes, at a=1, f(x) ≤0 on [1, ∞). For a>1, as shown, the function has a positive maximum, so f(x) exceeds 0. For a<1, the function also satisfies f(x) ≤0. But the problem asks for the maximum value of a such that f(x) ≤0. Therefore, the maximum a is 1.But wait, let me check if when a is less than 1, f(x) is ≤0. For example, take a=0.5. Then f(x)=ln x -x² +1. At x=1, f(1)=0 -1 +1=0. Then, since the function is decreasing on [1, ∞), it goes to -infty. So yes, f(x) ≤0. So any a ≤1 would satisfy f(x) ≤0 on [1, ∞). Therefore, the maximum a is 1.Therefore, the answer is 1.Wait, but why in part 2, when a=0.5, the maximum is 0? Because the maximum is at x=1, which is 0, and then it decreases. So f(x) is always ≤0. So for a≤1, f(x) ≤0. For a>1, not. Therefore, the maximum value of a is 1.Summary:1. For ( a=1 ), the function is decreasing on ( [1, infty) ).2. For ( a>0 ), the maximum of ( f(x) ) on ( [1, infty) ) is ( a ln a - a +1 ) if ( a geq1 ), otherwise 0.3. The maximum value of ( a ) for which ( f(x) leq0 ) on ( [1, infty) ) is (1).Final Answer1. boxed{[1, +infty)}2. The maximum value is (boxed{0}) when (0 < a < 1) and (boxed{a ln a - a + 1}) when (a geq 1).3. boxed{1}Note: For question 2, since the problem asks for the maximum value in terms of (a), a piecewise function is appropriate. However, depending on the required format, sometimes both cases are presented separately. The answer above reflects both possibilities as per the original problem's structure.</think>Given the function ( f(x) = 2aln x - x^2 + 1 ):1. If ( a = 1 ), find the interval where the function ( f(x) ) is monotonically decreasing. The derivative of ( f(x) ) when ( a = 1 ) is: [ f'(x) = frac{2}{x} - 2x ] Setting ( f'(x) leq 0 ): [ frac{2}{x} - 2x leq 0 implies 2 - 2x^2 leq 0 implies x^2 geq 1 implies x geq 1 ] Therefore, the function is monotonically decreasing on the interval (boxed{[1, +infty)}).2. If ( a > 0 ), find the maximum value of the function ( f(x) ) on the interval ([1, +infty)). The derivative of ( f(x) ) is: [ f'(x) = frac{2a}{x} - 2x ] Setting ( f'(x) = 0 ) gives the critical point ( x = sqrt{a} ). - For ( a geq 1 ), the critical point ( x = sqrt{a} ) is within the interval ([1, +infty)). Evaluating ( f(x) ) at this point: [ f(sqrt{a}) = 2a ln sqrt{a} - (sqrt{a})^2 + 1 = a ln a - a + 1 ] - For ( 0 < a < 1 ), the critical point ( x = sqrt{a} ) is outside the interval ([1, +infty)), so the maximum value occurs at ( x = 1 ): [ f(1) = 2a ln 1 - 1^2 + 1 = 0 ] Therefore, the maximum value is (boxed{0}) when (0 < a < 1) and (boxed{a ln a - a + 1}) when (a geq 1).3. If ( f(x) leq 0 ) holds true on the interval ([1, +infty)), find the maximum value of ( a ). From part 2, the maximum value of ( f(x) ) on ([1, +infty)) is ( a ln a - a + 1 ) for ( a geq 1 ) and 0 for ( 0 < a < 1 ). For ( f(x) leq 0 ) to hold, we need: [ a ln a - a + 1 leq 0 ] Solving this, we find that ( a = 1 ) is the maximum value because for ( a > 1 ), the expression ( a ln a - a + 1 ) becomes positive. For ( a leq 1 ), the maximum value is 0, which satisfies ( f(x) leq 0 ). Therefore, the maximum value of ( a ) is (boxed{1}).]

🔑:1. For a=1, we have the function f(x)=2ln x-x^{2}+1. To find where it is monotonically decreasing, we differentiate f(x) with respect to x: [ f'(x) = frac{d}{dx}(2ln x-x^{2}+1) = frac{2}{x} - 2x. ] Simplifying, we get: [ f'(x) = frac{2-2x^{2}}{x} = frac{-2(x^{2}-1)}{x}. ] Since x > 0, for f'(x) 0, which simplifies to x > 1. Thus, the function f(x) is monotonically decreasing in the interval (1,+infty). Therefore, the interval where the function is monotonically decreasing is boxed{(1,+infty)}.2. For a > 0, the derivative of f(x) is given by: [ f'(x) = frac{d}{dx}(2aln x-x^{2}+1) = frac{2a}{x} - 2x = frac{-2(x^{2}-a)}{x}. ] Setting f'(x) = 0 gives x^{2}-a = 0, which leads to x_{1}=sqrt{a} and x_{2}=-sqrt{a}. Since x > 0, we only consider x_{1}=sqrt{a}. Analyzing the sign of f'(x) and the value of f(x): - For 0 1, f(x) increases until x=sqrt{a} and then decreases. The maximum value of f(x) on [1,+infty) is at x=sqrt{a}, which is f(sqrt{a})=aln a-a+1. Therefore, the maximum value of the function f(x) on the interval [1,+infty) is boxed{f(1)=0} for 0 1.3. To find the maximum value of a when f(x) leqslant 0 on [1,+infty): - For 0 1, there exists x=sqrt{a} where f(x) > 0, contradicting the condition f(x) leqslant 0. Thus, the maximum value of a for which f(x) leqslant 0 on [1,+infty) is boxed{1}.

❓:A bug walks all day and sleeps all night. On the first day, it starts at point O, faces east, and walks a distance of 5 units due east. Each night the bug rotates 60^circ counterclockwise. Each day it walks in this new direction half as far as it walked the previous day. The bug gets arbitrarily close to the point P. Then OP^2=tfrac{m}{n}, where m and n are relatively prime positive integers. Find m+n.

🧠:[<think>Okay, let me try to figure out this bug problem. So, the bug starts at point O, faces east, walks 5 units east on the first day. Each night, it rotates 60 degrees counterclockwise, and each day it walks half the distance it did the previous day in this new direction. We need to find the square of the distance from O to the point P that the bug approaches, expressed as a reduced fraction m/n, and then find m + n.Hmm. Alright, let's break this down. The bug's movement seems to form a geometric series, but with direction changes each time. Since the directions are changing by 60 degrees each time, and the distances are halved each day, this might be a vector problem where each subsequent movement is a vector rotated by 60 degrees and scaled by half.So, starting at point O, facing east, the first movement is 5 units east. Then, each subsequent movement is half the previous length and 60 degrees counterclockwise from the previous direction. The bug does this infinitely, getting closer and closer to point P. We need to sum all these vectors to find the final position P, and then compute OP squared.Let me recall that when dealing with such problems, complex numbers might be helpful. Because each movement can be represented as a complex number, and rotating by 60 degrees is equivalent to multiplying by e^(iπ/3) in complex plane.Yes, so if we model the plane as the complex plane, with O at the origin, and east as the positive real axis. Then each step can be represented as a complex number. The first step is 5 units east, which is 5 + 0i. Then the next step is (5/2) units in a direction 60 degrees north of east, which is 5/2 * (cos 60° + i sin 60°). Then the next step is (5/4) units in a direction 120 degrees from east, which is 5/4 * (cos 120° + i sin 120°), etc.So, the total displacement OP is the sum of the infinite series: 5 + (5/2)(cos 60° + i sin 60°) + (5/4)(cos 120° + i sin 120°) + (5/8)(cos 180° + i sin 180°) + ... and so on.This is a geometric series where each term is multiplied by (1/2)(cos 60° + i sin 60°). Let's confirm that.First term: a = 5. The common ratio r would be (1/2)(cos 60° + i sin 60°). Since each subsequent term is half the length and rotated 60 degrees. So yes, the ratio between terms is multiplying by (1/2) and rotating 60°, which in complex numbers is multiplication by (1/2) e^(iπ/3).Therefore, the total displacement is the sum of the infinite geometric series with first term a = 5 and common ratio r = (1/2)(cos 60° + i sin 60°). The formula for the sum of an infinite geometric series is a / (1 - r), provided that |r| < 1. Here, |r| = 1/2 < 1, so it converges.Therefore, the total displacement OP is 5 / (1 - r), where r = (1/2)(cos 60° + i sin 60°). Let's compute that.First, compute cos 60° and sin 60°: cos 60° = 0.5, sin 60° = (√3)/2. So r = (1/2)(0.5 + i (√3)/2) = (1/2)(1/2 + i (√3)/2) = 1/4 + i (√3)/4.Therefore, 1 - r = 1 - (1/4 + i (√3)/4) = 3/4 - i (√3)/4. Then, OP is 5 divided by (3/4 - i (√3)/4). To compute this division, we can multiply numerator and denominator by the complex conjugate of the denominator.The complex conjugate of 3/4 - i (√3)/4 is 3/4 + i (√3)/4. So,OP = 5 * [ (3/4 + i (√3)/4) / ( (3/4)^2 + ( (√3)/4 )^2 ) ]First compute the denominator: (9/16) + (3/16) = 12/16 = 3/4. So denominator is 3/4.Therefore, OP = 5 * [ (3/4 + i (√3)/4 ) / (3/4) ] = 5 * [ (3/4)/(3/4) + i (√3)/4 / (3/4) ) ] = 5 * [ 1 + i (√3)/3 ].So OP = 5 + 5i (√3)/3. So in terms of coordinates, the real part is 5 and the imaginary part is 5√3/3. Therefore, point P has coordinates (5, 5√3/3). Wait, hold on. Wait, no. Wait, let me check that again.Wait, when we start facing east, the first step is east, so that's along the positive x-axis. Then each subsequent direction is 60 degrees counterclockwise. So the first direction is 0 degrees (east), then 60 degrees, then 120 degrees, then 180 degrees, etc. So each movement is in direction 60(n-1) degrees on the nth day.But when we modeled the problem using complex numbers, each term is multiplied by r = (1/2)(cos 60° + i sin 60°). So starting with the first term 5, then the second term is 5 * r, third term is 5 * r^2, etc. So each subsequent term is multiplied by r, which includes both the scaling by 1/2 and the rotation by 60°. Therefore, the direction of each step is 0°, 60°, 120°, etc., which matches the problem description. So the coordinates would be the sum of all these steps.But when I computed the sum, I found OP = 5 / (1 - r) = 5 / (3/4 - i√3/4) = 5 * (3/4 + i√3/4) / (3/4). Wait, but (3/4 - i√3/4) multiplied by its conjugate (3/4 + i√3/4) is ( (3/4)^2 + ( √3 /4 )^2 ) = 9/16 + 3/16 = 12/16 = 3/4. So denominator is 3/4. Therefore, the division gives 5 * (3/4 + i√3/4 ) / (3/4) ) = 5*(1 + i√3/3 ) = 5 + (5√3/3)i. So in rectangular coordinates, the x-coordinate is 5 and the y-coordinate is 5√3/3. Wait, but that can't be right, because the first step is 5 east, then the next step is 5/2 in direction 60°, which has both x and y components. So the total x-coordinate should be more than 5, and there should be some y-component. Wait, but according to this, the x-coordinate is 5, and the y-coordinate is 5√3/3. Hmm. Maybe that's correct?Wait, let's check with another approach. Let's model the first few terms:First term: 5 east. Position after first day: (5, 0).Second day: direction is 60°, distance is 5/2. The displacement is (5/2 cos 60°, 5/2 sin 60°) = (5/2 * 0.5, 5/2 * (√3)/2) = (5/4, 5√3/4). So new position: (5 + 5/4, 0 + 5√3/4) = (25/4, 5√3/4).Third day: direction is 120°, distance is 5/4. Displacement: (5/4 cos 120°, 5/4 sin 120°) = (5/4 * (-0.5), 5/4 * (√3)/2) = (-5/8, 5√3/8). New position: (25/4 - 5/8, 5√3/4 + 5√3/8) = (50/8 - 5/8, 10√3/8 + 5√3/8) = (45/8, 15√3/8).Fourth day: direction is 180°, distance is 5/8. Displacement: (-5/8, 0). New position: (45/8 - 5/8, 15√3/8) = (40/8, 15√3/8) = (5, 15√3/8).Fifth day: direction is 240°, distance is 5/16. Displacement: (5/16 cos 240°, 5/16 sin 240°) = (5/16*(-0.5), 5/16*(-√3)/2) = (-5/32, -5√3/32). New position: (5 - 5/32, 15√3/8 - 5√3/32) = (160/32 - 5/32, 60√3/32 - 5√3/32) = (155/32, 55√3/32).Sixth day: direction 300°, distance 5/32. Displacement: (5/32 cos 300°, 5/32 sin 300°) = (5/32*0.5, 5/32*(-√3)/2) = (5/64, -5√3/64). New position: (155/32 + 5/64, 55√3/32 - 5√3/64) = (310/64 + 5/64, 110√3/64 - 5√3/64) = (315/64, 105√3/64).Hmm. So the x-coordinate after six days is 315/64 ≈ 4.921875, and y-coordinate is 105√3/64 ≈ 2.8619. But according to the formula from the infinite series, we should approach (5, 5√3/3). Let's check what 5√3/3 is approximately: √3 ≈ 1.732, so 5*1.732/3 ≈ 2.887. That's close to the y-coordinate after six days. Similarly, the x-coordinate after six days is 315/64 ≈ 4.9219, which is close to 5. So maybe the formula is correct.But why does the infinite sum formula give (5, 5√3/3) instead of converging to something else? Let me check the calculations again.So, the total displacement OP is the sum S = 5 + (5/2)(cos60° + i sin60°) + (5/4)(cos120° + i sin120°) + ... which is a geometric series with first term a = 5 and common ratio r = (1/2)(cos60° + i sin60°). So S = a / (1 - r) = 5 / (1 - r). Then, substituting r = (1/2)(cos60° + i sin60°) = (1/2)(1/2 + i√3/2) = 1/4 + i√3/4. Therefore, 1 - r = 1 - 1/4 - i√3/4 = 3/4 - i√3/4.So S = 5 / (3/4 - i√3/4). To compute this, multiply numerator and denominator by the complex conjugate of the denominator, which is 3/4 + i√3/4:S = 5*(3/4 + i√3/4) / [ (3/4)^2 + (√3/4)^2 ]Compute denominator: (9/16) + (3/16) = 12/16 = 3/4.Therefore, S = 5*(3/4 + i√3/4) / (3/4) = 5*(1 + i√3/3) = 5 + (5√3/3)i. So yes, this is 5 in the real (east) direction and 5√3/3 in the imaginary (north) direction. Therefore, the coordinates of P are (5, 5√3/3). Therefore, OP squared is 5^2 + (5√3/3)^2.Compute that:5^2 = 25(5√3/3)^2 = (25*3)/9 = 75/9 = 25/3Therefore, OP squared is 25 + 25/3 = 75/3 + 25/3 = 100/3. Therefore, OP² = 100/3, so m = 100, n = 3. They are coprime, so m + n = 103.But wait, let's check if this answer makes sense. If after each cycle, the bug approaches (5, 5√3/3), then OP² would be 25 + (25*3)/9 = 25 + 25/3 = 100/3 ≈ 33.333. Let me check with the position after six days: (315/64 ≈ 4.9219, 105√3/64 ≈ 2.8619). Then OP² would be approximately (4.9219)^2 + (2.8619)^2 ≈ 24.225 + 8.191 ≈ 32.416. Which is close to 100/3 ≈ 33.333, so maybe converging towards that. So seems plausible.But wait, when I computed the position after the fourth day, it was (5, 15√3/8). Then OP squared is 25 + (225*3)/64 = 25 + 675/64 = 25 + 10.546875 ≈ 35.546875. Wait, that's higher than 33.333. Then fifth day: (155/32 ≈ 4.84375, 55√3/32 ≈ 2.910). Then OP squared is (155/32)^2 + (55√3/32)^2. Compute:155/32 ≈ 4.84375. Squared ≈ 23.46. 55√3/32 ≈ 2.910. Squared ≈ 8.4681. Total ≈ 31.928. Then sixth day: 315/64 ≈ 4.9219, 105√3/64 ≈ 2.8619. Squared ≈ 24.22 + 8.19 ≈ 32.41. So the OP squared is oscillating around 33.333, getting closer. So maybe 100/3 is the correct answer.But let me think again. Maybe there's a miscalculation in the complex number approach.Wait, another way to model this: The displacement is a vector sum. Each step is a vector with magnitude 5*(1/2)^{n-1} and direction 60°*(n-1). So the total displacement is the sum from n=1 to infinity of 5*(1/2)^{n-1} [cos(60°*(n-1)) + i sin(60°*(n-1))].This is indeed a geometric series with first term 5 and common ratio (1/2)(cos60° + i sin60°). So the formula should be correct.Alternatively, to compute this sum in terms of real and imaginary parts, we can separate the series into real and imaginary components.Let me write S = sum_{n=0}^infty 5*(1/2)^n [cos(60°*n) + i sin(60°*n)]But since the first term is n=0: 5*(1/2)^0 [cos0 + i sin0] = 5*1*[1 + 0i] = 5.But in the problem statement, the first day is 5 units east (n=1). Wait, maybe indexing is different. Wait, maybe n starts at 1? Let me check.Wait, the first day is 5 units east: that would correspond to n=1, direction 0°, which is 60°*(1-1) = 0°. Then n=2, direction 60°, n=3, direction 120°, etc. So the series is sum_{n=1}^infty 5*(1/2)^{n-1} [cos(60°*(n-1)) + i sin(60°*(n-1))].Which can be rewritten as 5 * sum_{k=0}^infty (1/2)^k [cos(60°k) + i sin(60°k)] where k = n -1.So S = 5 * sum_{k=0}^infty [ (1/2) (cos60° + i sin60°) ]^kWhich is 5 / (1 - (1/2)(cos60° + i sin60°)) ), same as before.Therefore, the formula is correct, and the result is OP = 5 + (5√3)/3 i. So OP squared is 25 + (25*3)/9 = 25 + 25/3 = 100/3. Thus, m = 100, n = 3, so m + n = 103.Therefore, the answer is boxed{103}.Final Answerboxed{103}</think>The bug starts at point ( O ) and faces east. On the first day, it walks 5 units east. Each subsequent day, it walks half the distance of the previous day in a direction rotated 60° counterclockwise. This forms a geometric series with complex number representation.1. First term: ( 5 ) units east (represented as ( 5 ) in the complex plane).2. Common ratio: ( frac{1}{2} left( cos 60^circ + i sin 60^circ right) = frac{1}{4} + i frac{sqrt{3}}{4} ).The total displacement ( OP ) is the sum of the infinite geometric series:[ S = frac{5}{1 - left( frac{1}{4} + i frac{sqrt{3}}{4} right)} ]3. Denominator calculation:[ 1 - left( frac{1}{4} + i frac{sqrt{3}}{4} right) = frac{3}{4} - i frac{sqrt{3}}{4} ]4. Complex conjugate of the denominator: ( frac{3}{4} + i frac{sqrt{3}}{4} ).5. Multiply numerator and denominator by the conjugate:[ S = 5 cdot frac{frac{3}{4} + i frac{sqrt{3}}{4}}{left( frac{3}{4} right)^2 + left( frac{sqrt{3}}{4} right)^2} = 5 cdot frac{frac{3}{4} + i frac{sqrt{3}}{4}}{frac{3}{4}} = 5 left( 1 + i frac{sqrt{3}}{3} right) ]6. Resulting coordinates:[ OP = 5 + i frac{5sqrt{3}}{3} ]7. Distance squared:[ OP^2 = 5^2 + left( frac{5sqrt{3}}{3} right)^2 = 25 + frac{75}{9} = 25 + frac{25}{3} = frac{75}{3} + frac{25}{3} = frac{100}{3} ]Thus, ( OP^2 = frac{100}{3} ), and ( m + n = 100 + 3 = 103 ).[boxed{103}],<think>Okay, let's try to tackle this problem. So, there's a bug that starts at point O, faces east, walks 5 units east on the first day. Each night, it rotates 60 degrees counterclockwise, and each day it walks half the distance it walked the previous day in this new direction. The question is asking for the square of the distance from the origin to the point P that the bug approaches, and then to find m + n where that squared distance is expressed as a reduced fraction m/n.Hmm. Alright, let me break this down. So, the bug is making a sequence of moves, each time rotating 60 degrees counterclockwise and walking half the distance. So this is a geometric series, but in the plane with complex numbers or vectors, right? Since each movement is in a different direction, we need to model the bug's position as the sum of vectors, each rotated by 60 degrees and scaled by half each time.First, let's recall that rotating a vector by 60 degrees counterclockwise can be represented in complex numbers by multiplying by e^(iπ/3) or in coordinates using rotation matrices. Since we might need to sum these vectors, working with complex numbers might make this easier. Let me try that approach.Let me assign the directions to the complex plane. The bug starts facing east, which is the positive real axis. So, east is the direction of 1 in complex numbers. Each subsequent direction is a 60-degree rotation counterclockwise, which would be multiplying the current direction by e^(iπ/3). Since each day the direction changes by another 60 degrees, the direction on day n is (e^(iπ/3))^(n-1).But also, each day the distance walked halves. So on day 1, it walks 5 units east. On day 2, it walks 5*(1/2) units in the direction 60 degrees north of east. On day 3, 5*(1/2)^2 units in the direction 120 degrees from east, and so on.Therefore, the total displacement from the origin will be the sum of these vectors. Let me formalize this.Let’s denote the displacement on day k as vector v_k. Then:v_1 = 5 * e^(i*0°)v_2 = 5*(1/2) * e^(i*60°)v_3 = 5*(1/2)^2 * e^(i*120°)...v_k = 5*(1/2)^(k-1) * e^(i*60°*(k-1))Therefore, the total displacement P is the sum from k=1 to infinity of v_k. So:P = Σ_{k=1}^∞ 5*(1/2)^{k-1} * e^{i*60°*(k-1)}This is a geometric series in complex numbers. The common ratio between terms is (1/2) * e^(i*60°). Let me verify that. Each term is multiplied by (1/2) * e^(i*60°) from the previous term. So:v_{k+1} = v_k * (1/2) * e^(i*60°)Yes, that's correct. So the first term a = v_1 = 5, and the common ratio r = (1/2) * e^(i*π/3). So the sum of an infinite geometric series is a / (1 - r), provided |r| < 1. Let's check the magnitude of r:|r| = |(1/2) * e^(i*π/3)| = 1/2 < 1. So the series converges.Therefore, P = 5 / (1 - (1/2)e^{iπ/3})Now, we need to compute this complex number P and then find its magnitude squared, which is OP^2. Then express that as m/n in reduced terms.Alright, so let's compute 1 - (1/2)e^{iπ/3} first. Let me write e^{iπ/3} in rectangular form. e^{iπ/3} is cos(π/3) + i sin(π/3) = 0.5 + i*(√3)/2. Therefore, (1/2)e^{iπ/3} is (1/2)(0.5 + i*(√3)/2) = 0.25 + i*(√3)/4.Therefore, 1 - (1/2)e^{iπ/3} = 1 - 0.25 - i*(√3)/4 = 0.75 - i*(√3)/4.So, P = 5 / (0.75 - i*(√3)/4)To compute this reciprocal, we can multiply numerator and denominator by the complex conjugate of the denominator to rationalize.The complex conjugate of 0.75 - i*(√3)/4 is 0.75 + i*(√3)/4.Therefore:P = 5 * (0.75 + i*(√3)/4) / [ (0.75)^2 + ( (√3)/4 )^2 ]Compute the denominator first:(0.75)^2 = (3/4)^2 = 9/16( (√3)/4 )^2 = (3)/16So denominator = 9/16 + 3/16 = 12/16 = 3/4Therefore, denominator is 3/4, so:P = 5 * (0.75 + i*(√3)/4) / (3/4) = 5 * ( (3/4) + i*(√3)/4 ) * (4/3) )Simplify:Multiply by 4/3: (3/4)*(4/3) = 1, so the 4/3 cancels the 3/4. So:P = 5 * (1 + i*(√3)/3 )Wait, let me check that step again.Wait, 0.75 is 3/4, so:Numerator after multiplying by conjugate: (3/4 + i√3/4)Denominator: 3/4So dividing (3/4 + i√3/4) by (3/4) is equivalent to multiplying numerator and denominator by 4/3:( (3/4 + i√3/4 ) * 4/3 ) / ( (3/4) * 4/3 ) ) = ( (3/4 *4/3) + i√3/4 *4/3 ) / 1 = (1 + i√3/3 )Therefore, P = 5 * (1 + i√3/3 )Thus, P in complex form is 5*(1 + i√3/3 ). To find the magnitude squared, we can compute |P|^2.Since P = a + ib, |P|^2 = a^2 + b^2.Compute a and b:a = 5*1 = 5b = 5*(√3)/3 = (5√3)/3Therefore, |P|^2 = (5)^2 + (5√3/3)^2 = 25 + (25*3)/9 = 25 + 75/9 = 25 + 25/3 = (75 + 25)/3 = 100/3Wait, hold on. Wait, (5√3/3)^2 is 25*3/9, which is 75/9 = 25/3. So 25 + 25/3 = 75/3 + 25/3 = 100/3. So |P|^2 = 100/3. Therefore, OP^2 = 100/3, so m = 100, n = 3, which are coprime. Hence, m + n = 103.Wait, but let me double-check the steps in case I messed up.So, the key steps:1. Recognize the movement as a geometric series with complex ratio.2. Sum the series by using the formula a/(1 - r), leading to 5 / (1 - (1/2)e^{iπ/3}).3. Convert the denominator into rectangular form and compute the reciprocal by multiplying numerator and denominator by the conjugate.4. Simplify to find P = 5*(1 + i√3/3).5. Calculate |P|^2 as 5^2 + (5√3/3)^2 = 25 + (25*3)/9 = 25 + 25/3 = 100/3.Yes, that seems right. Wait, but let me check step 2 again. So the first term a is 5, and the ratio r is (1/2)e^{iπ/3}. Therefore, sum is a / (1 - r) = 5 / (1 - (1/2)e^{iπ/3} ), which is correct.Then when we converted e^{iπ/3} to rectangular, that's cos(60°) + i sin(60°) = 0.5 + i(√3/2). Then multiplied by 1/2 gives 0.25 + i√3/4, so 1 - that is 0.75 - i√3/4. Then taking the reciprocal as done.Then multiplying numerator and denominator by conjugate, computing denominator as 3/4, and then the numerator becomes 5*(0.75 + i√3/4 ) divided by 3/4, which becomes 5*( (0.75 + i√3/4 ) * (4/3) ) = 5*( (3/4*4/3) + i√3/4*4/3 ) = 5*(1 + i√3/3 ). So P = 5 + (5√3/3)i. Then squared magnitude is 25 + (25*3)/9 = 25 + 75/9 = 25 + 25/3 = 100/3. So yes, m = 100, n = 3. So m + n = 103.Therefore, the answer is 103.But let me check if there is an alternative approach to confirm.Alternatively, we can model each step as vectors in the plane.Each displacement can be represented in terms of east and north components.First day: 5 units east. So x1 = 5, y1 = 0.Second day: walks 5*(1/2) = 2.5 units in a direction 60 degrees from east. So the angle with the east axis is 60 degrees. The components are 2.5 cos(60°) east and 2.5 sin(60°) north. cos(60°) = 0.5, sin(60°) = √3/2. So x2 = 2.5 * 0.5 = 1.25, y2 = 2.5 * √3/2 ≈ 2.16506...Third day: direction is 60 + 60 = 120 degrees from east. Walks 2.5*(1/2) = 1.25 units. So components: 1.25 cos(120°) and 1.25 sin(120°). Cos(120°) = -0.5, sin(120°) = √3/2. So x3 = 1.25*(-0.5) = -0.625, y3 = 1.25*(√3/2) ≈ 1.08253...Fourth day: direction 180 degrees from east (i.e., west), walks 0.625 units. So x4 = 0.625*cos(180°) = -0.625, y4 = 0.625*sin(180°) = 0.Fifth day: direction 240 degrees from east, walks 0.3125 units. Components: 0.3125 cos(240°) and 0.3125 sin(240°). Cos(240°) = cos(180° + 60°) = -cos(60°) = -0.5, sin(240°) = sin(180° + 60°) = -sin(60°) = -√3/2. So x5 = 0.3125*(-0.5) = -0.15625, y5 = 0.3125*(-√3/2) ≈ -0.27063...Sixth day: direction 300 degrees from east, walks 0.15625 units. Cos(300°) = cos(-60°) = 0.5, sin(300°) = sin(-60°) = -√3/2. So x6 = 0.15625*0.5 = 0.078125, y6 = 0.15625*(-√3/2) ≈ -0.13531...Seventh day: direction 360° = 0°, back to east, walks 0.078125 units. So x7 = 0.078125, y7 = 0.And so on. Each time, the direction cycles every 6 days? But since each day the angle increases by 60°, the directions cycle every 6 days, but the distance is halved each day.But to compute the total displacement, we need to sum all these vectors. However, since each term is a vector with a magnitude decreasing by half each time and rotated 60°, this seems a geometric series in complex plane as I did earlier.But perhaps we can model this as the sum of x-components and y-components separately.Let me try that.Let’s denote each displacement as (xk, yk), where xk is the east component and yk is the north component.From the above, the first few terms are:Day 1: (5, 0)Day 2: (5*(1/2)*cos(0° + 60°*1), 5*(1/2)*sin(60°*1))Wait, wait. Wait, direction on day k is 60°*(k - 1). So general term on day k:xk = 5*(1/2)^{k - 1} * cos(60°*(k - 1))yk = 5*(1/2)^{k - 1} * sin(60°*(k - 1))Therefore, total x = Σ_{k=1}^∞ xk = 5 Σ_{k=0}^∞ (1/2)^k cos(60°*k )Similarly, total y = Σ_{k=1}^∞ yk = 5 Σ_{k=0}^∞ (1/2)^k sin(60°*k )Wait, shifting index: let’s let n = k - 1, then when k=1, n=0. So:Total x = 5 Σ_{n=0}^∞ (1/2)^n cos(60°*n )Total y = 5 Σ_{n=0}^∞ (1/2)^n sin(60°*n )These are the real and imaginary parts of the complex series 5 Σ_{n=0}^∞ (1/2)^n e^{i60°n }, which is the same as the previous approach, so we get the same result.Therefore, using the formula for the sum of such series:If S = Σ_{n=0}^∞ r^n e^{iθn }, then S = 1 / (1 - r e^{iθ} ), provided |r e^{iθ}| < 1.Therefore, the total displacement is 5 * S, where S = 1 / (1 - (1/2)e^{i60°} ), same as before.Therefore, the result is the same. So, the squared magnitude is 100/3, answer is 103.Alternatively, perhaps there's a way to compute the sum of these cosine and sine terms without complex numbers. Let me try that.For the x-component:Total x = 5 Σ_{n=0}^∞ (1/2)^n cos(60°n )Similarly for y-component.There is a formula for such sums. For example, Σ_{n=0}^infty r^n cos(nθ) = (1 - r cosθ)/(1 - 2 r cosθ + r^2 )Similarly, Σ_{n=0}^infty r^n sin(nθ) = (r sinθ)/(1 - 2 r cosθ + r^2 )So, in our case, r = 1/2, θ = 60°, which is π/3 radians.Compute total x:Σ cos(nθ) (1/2)^n = [1 - (1/2) cosθ ] / [1 - 2*(1/2) cosθ + (1/2)^2 ]Let me compute denominator:1 - 2*(1/2) cosθ + (1/2)^2 = 1 - cosθ + 1/4 = 5/4 - cosθNumerator: 1 - (1/2) cosθTherefore, Σ cos(nθ)(1/2)^n = [1 - (1/2) cosθ] / [5/4 - cosθ]Similarly, Σ sin(nθ)(1/2)^n = [ (1/2) sinθ ] / [5/4 - cosθ ]So, total x = 5 * [1 - (1/2) cosθ ] / [5/4 - cosθ ]Total y = 5 * [ (1/2) sinθ ] / [5/4 - cosθ ]Let’s plug θ = 60°, so cosθ = 0.5, sinθ = √3/2.Compute numerator for total x:1 - (1/2)(0.5) = 1 - 0.25 = 0.75Denominator:5/4 - 0.5 = 5/4 - 2/4 = 3/4Therefore, total x = 5 * (0.75) / (0.75) = 5 * 1 = 5Similarly, numerator for total y:(1/2)(√3/2) = √3/4Denominator same: 3/4Total y = 5 * (√3/4) / (3/4) = 5 * (√3/4) * (4/3) ) = 5 * √3 / 3Therefore, total x = 5, total y = (5√3)/3So, squared distance is 5^2 + (5√3/3)^2 = 25 + (25*3)/9 = 25 + 75/9 = 25 + 25/3 = 75/3 + 25/3 = 100/3.Same result as before. So, that confirms it. Therefore, the answer is 100/3, so m + n = 103.Therefore, the answer is boxed{103}Final Answerboxed{103}</think>The bug starts at point ( O ) and faces east. Each day, it walks half the distance it walked the previous day in a new direction, which is rotated ( 60^circ ) counterclockwise from the previous day. We need to find the square of the distance from the origin to the point ( P ) that the bug approaches.1. Geometric Series in Complex Plane: - The displacement on day ( k ) is ( 5 left( frac{1}{2} right)^{k-1} e^{i cdot 60^circ (k-1)} ). - The total displacement ( P ) is the sum of this geometric series: [ P = sum_{k=1}^{infty} 5 left( frac{1}{2} right)^{k-1} e^{i cdot 60^circ (k-1)} ] - The common ratio ( r = frac{1}{2} e^{i cdot 60^circ} ) has magnitude ( frac{1}{2} ), so the series converges.2. Sum of the Series: - The sum of the series is given by ( frac{5}{1 - frac{1}{2} e^{i cdot 60^circ}} ). - Convert ( e^{i cdot 60^circ} ) to rectangular form: ( cos(60^circ) + i sin(60^circ) = frac{1}{2} + i frac{sqrt{3}}{2} ). - The denominator becomes ( 1 - frac{1}{2} left( frac{1}{2} + i frac{sqrt{3}}{2} right) = frac{3}{4} - i frac{sqrt{3}}{4} ).3. Rationalizing the Denominator: - Multiply numerator and denominator by the conjugate ( frac{3}{4} + i frac{sqrt{3}}{4} ): [ P = 5 cdot frac{frac{3}{4} + i frac{sqrt{3}}{4}}{left( frac{3}{4} right)^2 + left( frac{sqrt{3}}{4} right)^2} = 5 cdot frac{frac{3}{4} + i frac{sqrt{3}}{4}}{frac{3}{4}} = 5 left( 1 + i frac{sqrt{3}}{3} right) ]4. Magnitude Squared: - The coordinates of ( P ) are ( 5 ) (real part) and ( frac{5sqrt{3}}{3} ) (imaginary part). - The magnitude squared is: [ OP^2 = 5^2 + left( frac{5sqrt{3}}{3} right)^2 = 25 + frac{25 cdot 3}{9} = 25 + frac{75}{9} = frac{75}{3} + frac{25}{3} = frac{100}{3} ]Thus, ( OP^2 = frac{100}{3} ), and ( m + n = 100 + 3 = 103 ). The final answer is (boxed{103}).]

🔑:We notice that the moves cycle every 6 moves, so we plot out the first 6 moves on the coordinate grid with point O as the origin. We will keep a tally of the x-coordinate and y-coordinate separately. Then, we will combine them and account for the cycling using the formula for an infinite geometric series.First move: The ant moves right 5. Second move: We use properties of a 30-60-90 triangle to get frac{5}{4} right, frac{5sqrt{3}}{4} up. Third move: frac{5}{8} left, frac{5sqrt{3}}{8} up.Fourth move: frac{5}{8} left.Fifth move: frac{5}{32} left, frac{5sqrt{3}}{32} down.Sixth move: frac{5}{64} right, frac{5sqrt{3}}{64} down.Total of x-coordinate: 5 + frac{5}{4} - frac{5}{8} - frac{5}{8} - frac{5}{32} + frac{5}{64} = frac{315}{64}.Total of y-coordinate: 0 + frac{5sqrt{3}}{4} + frac{5sqrt{3}}{8} + 0 - frac{5sqrt{3}}{32} - frac{5sqrt{3}}{64} = frac{105sqrt{3}}{64}.After this cycle of six moves, all moves repeat with a factor of (frac{1}{2})^6 = frac{1}{64}. Using the formula for a geometric series, multiplying each sequence by frac{1}{1-frac{1}{64}} = frac{64}{63} will give us the point P.Now, knowing the initial x and y, we plug this into the geometric series formula (frac{a}{1-r}), and we get frac{315}{64} cdot frac{64}{63} = 5, frac{105sqrt{3}}{64} cdot frac{64}{63} = frac{5sqrt{3}}{3}.Therefore, the coordinates of point P are (5,frac{5sqrt{3}}{3}), so using the Pythagorean Theorem, OP^2 = frac{100}{3}, for an answer of boxed{103}.-molocyxu

❓:The side lengths of a scalene triangle are roots of the polynomial x^3-20x^2+131x-281.3. Find the square of the area of the triangle.

🧠:[<think>Alright, so I have this problem here where the side lengths of a scalene triangle are the roots of the polynomial (x^3 - 20x^2 + 131x - 281.3). I need to find the square of the area of this triangle. Hmm, okay. Let's think about how to approach this.First off, the polynomial is given as (x^3 - 20x^2 + 131x - 281.3). The roots of this polynomial are the sides of the triangle, which is scalene. So, all sides are distinct, which makes sense because it's a scalene triangle. The roots are a, b, c, the sides. So, the roots satisfy the relationships from Vieta's formulas. Let me recall Vieta's for a cubic equation: for (x^3 + px^2 + qx + r = 0), the sum of roots is -p, the sum of products two at a time is q, and the product is -r. Wait, but in our polynomial, the coefficient of (x^3) is positive 1, so the formula applies directly.So, for our polynomial (x^3 - 20x^2 + 131x - 281.3), the sum of the roots (a + b + c) would be 20. The sum of the products two at a time (ab + bc + ac) is 131, and the product of the roots (abc) is 281.3. So, that's a, b, c. Now, the area of a triangle when you know all three sides is given by Heron's formula. Heron's formula states that the area is (sqrt{s(s - a)(s - b)(s - c)}), where (s) is the semi-perimeter, ((a + b + c)/2). Since we need the square of the area, we can just compute (s(s - a)(s - b)(s - c)). That might be easier since we won't have to deal with the square root. So, let's denote the square of the area as (K^2 = s(s - a)(s - b)(s - c)). Given that, we need to compute this expression. But how do we compute this expression if we don't know the individual values of a, b, c? Well, we can express everything in terms of the sums and products of the roots, which we know from Vieta's formulas. Let's see.First, let's compute (s), which is ((a + b + c)/2 = 20/2 = 10). So, s = 10.Then, we need to compute the product (s(s - a)(s - b)(s - c)). Let's rewrite that as (10 times (10 - a)(10 - b)(10 - c)). So, if we can compute ((10 - a)(10 - b)(10 - c)), multiply by 10, and that will give us (K^2).So, the problem reduces to finding the value of ((10 - a)(10 - b)(10 - c)). Let's think about how to compute this.Expanding ((10 - a)(10 - b)(10 - c)), we can use the formula for expanding such products. The expansion of ((x - a)(x - b)(x - c)) is (x^3 - (a + b + c)x^2 + (ab + bc + ac)x - abc). Wait, that's exactly the original polynomial but evaluated at x = 10. Wait, let's see:If the original polynomial is (f(x) = x^3 - 20x^2 + 131x - 281.3), then ((x - a)(x - b)(x - c) = f(x)). Therefore, substituting x = 10 into f(x) would give us (10^3 - 20 times 10^2 + 131 times 10 - 281.3). Let me compute that:(10^3 = 1000)(20 times 10^2 = 20 times 100 = 2000)(131 times 10 = 1310)So, substituting:(1000 - 2000 + 1310 - 281.3 = (1000 - 2000) + (1310 - 281.3) = (-1000) + (1028.7) = 28.7).Therefore, ((10 - a)(10 - b)(10 - c) = f(10) = 28.7). Therefore, the square of the area is (10 times 28.7 = 287). But wait, that seems straightforward. Let me verify.Wait, but Heron's formula is (sqrt{s(s - a)(s - b)(s - c)}), so the square is (s(s - a)(s - b)(s - c)), which we have as 10 * 28.7 = 287. So, is the answer 287? But the problem says the polynomial has roots a, b, c, which are the sides of a triangle. So, before concluding, we need to check whether such a triangle exists. Because not all sets of three positive numbers can form a triangle. They have to satisfy the triangle inequality: the sum of any two sides must be greater than the third side.But given that it's stated as a scalene triangle, the problem must ensure that such a triangle exists. But maybe we need to confirm that. Let me see. The sides are the roots of the polynomial. Since the polynomial is cubic with real coefficients, the roots are either all real or one real and two complex conjugates. But since it's given that it's a triangle, the sides must be real and positive, so all three roots are real and positive. Also, since it's scalene, they are distinct. So, the polynomial must have three distinct positive real roots. Let's confirm that.First, we can check if the polynomial has three real roots. Let's compute the discriminant of the cubic. The discriminant D of a cubic equation (x^3 + ax^2 + bx + c = 0) is given by (D = 18abc - 4a^3c + a^2b^2 - 4b^3 - 27c^2). Wait, but our polynomial is (x^3 - 20x^2 + 131x - 281.3), so coefficients are:a = -20, b = 131, c = -281.3Wait, actually, the general form is (x^3 + px^2 + qx + r = 0), so in our case, p = -20, q = 131, r = -281.3.The discriminant formula for cubic (x^3 + px^2 + qx + r) is:Δ = 18pqr - 4p^3r + p^2q^2 - 4q^3 - 27r^2So plugging in:Δ = 18*(-20)*131*(-281.3) - 4*(-20)^3*(-281.3) + (-20)^2*(131)^2 - 4*(131)^3 - 27*(-281.3)^2This is going to be a bit complicated to compute, but maybe we can approximate. Alternatively, perhaps the polynomial has three real roots. Alternatively, maybe since the problem states it's a triangle, we can assume the roots are positive and satisfy triangle inequalities. But just to be safe, maybe we can check if f(10) is positive, but we already found that f(10) = 28.7. Wait, but f(10) = 28.7, which is positive, but how does that relate to the roots?Wait, actually, if we consider that the roots are a, b, c, then each of them is less than 10? Because when we compute (10 - a)(10 - b)(10 - c) = 28.7, which is positive. So, each of (10 - a), (10 - b), (10 - c) must be positive, meaning that a, b, c are all less than 10. But wait, the semi-perimeter is 10. If all sides are less than 10, then each side is less than the semi-perimeter. Wait, but in a triangle, each side must be less than the sum of the other two sides. Since the semi-perimeter is 10, each side must be less than 10 (since the sum of the other two sides would be 20 - a, so for triangle inequality, a < b + c = 20 - a => 2a < 20 => a < 10). So, yes, each side must be less than 10. So, (10 - a), etc., are positive, so their product is positive, which matches with f(10) being positive.So, that seems okay. Therefore, the calculation that the square of the area is 10 * 28.7 = 287. But 287 is a decimal. However, the polynomial coefficients are given with one decimal place (281.3). Maybe the answer is 287.0? But perhaps we need to check whether there is a miscalculation here. Let me verify the computation step by step.First, compute f(10):(10^3 - 20*10^2 + 131*10 - 281.3)Calculating each term:10^3 = 100020*10^2 = 20*100 = 2000131*10 = 1310281.3 is just 281.3So,1000 - 2000 = -1000-1000 + 1310 = 310310 - 281.3 = 28.7Yes, that's correct. So, f(10) = 28.7, so 10 * 28.7 = 287. So, the square of the area is 287, so the answer should be 287. However, 287 is not a decimal; perhaps the problem expects an exact value? Wait, the polynomial has coefficients with decimal points (281.3). So, maybe 281.3 is a decimal, so the answer is 287. But 287 can be written as 287.0, but since the input has one decimal place, maybe the answer is 287.0, but maybe it's a fraction? Let me check if 281.3 is a decimal that can be converted to a fraction. 281.3 is equal to 2813/10. Let's see:So, if the polynomial is (x^3 - 20x^2 + 131x - 2813/10), then when we compute f(10) = 1000 - 2000 + 1310 - 2813/10.Let me compute this using fractions to see if 28.7 is exact.Convert all terms to tenths:1000 = 10000/102000 = 20000/101310 = 13100/102813/10 is already in tenths.So,10000/10 - 20000/10 + 13100/10 - 2813/10 =(10000 - 20000 + 13100 - 2813)/10 =(10000 - 20000 = -10000; -10000 + 13100 = 3100; 3100 - 2813 = 287)/10 = 287/10 = 28.7So, yes, exactly 28.7. Therefore, 10 * 28.7 = 287. So, the square of the area is 287. Hence, the answer is 287, which can be written as boxed{287}Wait, but let me check again whether Heron's formula is applicable here. Heron's formula requires that the three sides satisfy the triangle inequality. Since it's given that it's a scalene triangle, they must satisfy the triangle inequalities, so we don't have to check, but just to be thorough, perhaps we can verify.But how can we verify the triangle inequalities without knowing the individual sides? Hmm. Since the sides are all less than 10 (as we saw earlier), and the sum of any two sides must be greater than the third. Since the sum of all three sides is 20, the sum of any two sides is 20 minus the third side. So, for sides a, b, c:a + b > c => 20 - c > c => 20 > 2c => c < 10. Which we already have.Similarly, the other inequalities would give the same result. Therefore, since all sides are less than 10, all triangle inequalities are satisfied. Therefore, the triangle is valid.Therefore, the calculation is correct, and the square of the area is indeed 287.But wait, just to make sure, let me recall another formula for the area of a triangle given the three sides. There's also the formula using the product of the sides and the radius of the circumscribed circle, but that might not help here. Alternatively, there is a formula related to the semi-perimeter and the roots. Wait, but we used Heron's formula, which is the standard way.Alternatively, I remember that in terms of the semi-perimeter and the roots, Heron's formula is the way to go, and we've expressed the product (10 - a)(10 - b)(10 - c) as f(10), which we calculated to be 28.7, leading to 10 * 28.7 = 287. So, seems correct.Alternatively, maybe using the formula for area in terms of the sides and the coefficients of the polynomial? But I think the method we used is the correct approach.Wait, but let me check an example. Suppose the sides are 5, 7, 8. Then the polynomial would be (x - 5)(x - 7)(x - 8) = x³ - 20x² + 131x - 280. Then the area squared would be s(s - a)(s - b)(s - c). Here, s = (5 + 7 + 8)/2 = 10. Then, s(s - a)(s - b)(s - c) = 10 * 5 * 3 * 2 = 300. So, the square of the area is 300. If we compute f(10) for this polynomial: 10³ -20*10² +131*10 -280 = 1000 - 2000 + 1310 -280 = (1000 -2000) + (1310 -280) = (-1000) + (1030) = 30. Then, 10 * 30 = 300, which matches. So, the method works here.But in our problem, the polynomial is x³ -20x² +131x -281.3. So, similar to the example, but with the constant term being 281.3 instead of 280. Therefore, when we compute f(10), we get 28.7 instead of 30, leading to 287 instead of 300. So, the method holds. Therefore, the answer should be 287.Wait, but in the example, the product (10 - a)(10 - b)(10 - c) = 30, leading to area squared 300. In our problem, it's 28.7, leading to 287, which is 30 - 1.3 = 28.7, and 30*10=300 vs 28.7*10=287. That seems plausible.Therefore, I think the answer is 287. However, since 281.3 is given as a decimal, which is 2813/10, the exact value would be 287/1, which is 287.0, but since the problem asks for the square of the area, it's 287. Therefore, the answer is boxed{287}Final Answerboxed{287}</think>The side lengths of a scalene triangle are the roots of the polynomial (x^3 - 20x^2 + 131x - 281.3). We need to find the square of the area of this triangle.Using Vieta's formulas, we know the sum of the roots (a + b + c = 20), the sum of the products of the roots taken two at a time (ab + bc + ca = 131), and the product of the roots (abc = 281.3).The semi-perimeter (s) of the triangle is:[s = frac{a + b + c}{2} = frac{20}{2} = 10]The area of the triangle can be found using Heron's formula, which states that the square of the area (K^2) is:[K^2 = s(s - a)(s - b)(s - c)]To find ((10 - a)(10 - b)(10 - c)), we evaluate the polynomial at (x = 10):[f(10) = 10^3 - 20 cdot 10^2 + 131 cdot 10 - 281.3]Calculating each term:[10^3 = 1000][20 cdot 10^2 = 2000][131 cdot 10 = 1310][1000 - 2000 + 1310 - 281.3 = -1000 + 1310 - 281.3 = 310 - 281.3 = 28.7]Thus, ((10 - a)(10 - b)(10 - c) = 28.7). Multiplying this by the semi-perimeter (s = 10) gives:[K^2 = 10 times 28.7 = 287]Therefore, the square of the area of the triangle is (boxed{287}).,<think>Alright, let's try to figure out how to find the square of the area of this scalene triangle whose side lengths are the roots of the polynomial (x^3 - 20x^2 + 131x - 281.3). Hmm, okay. So, first, the sides of the triangle are the roots of this cubic equation. That means if we call the roots (a), (b), and (c), then they satisfy the polynomial. Since it's a scalene triangle, all sides are of different lengths, so (a), (b), and (c) are distinct. To find the area, I remember Heron's formula, which requires the semi-perimeter and the sides. The formula is (Area = sqrt{s(s - a)(s - b)(s - c)}), where (s = frac{a + b + c}{2}). But since we need the square of the area, that would just be (s(s - a)(s - b)(s - c)). But here's the thing: the roots (a), (b), and (c) are given by the polynomial, so we can use Vieta's formulas to find their sum, product, and sum of products without actually solving for the roots. Let me recall Vieta's formulas for a cubic equation (x^3 + px^2 + qx + r = 0). The sum of roots is (-p), the sum of products two at a time is (q), and the product of roots is (-r). Wait, in our case, the polynomial is (x^3 - 20x^2 + 131x - 281.3). So comparing to the general form, (p = -20), (q = 131), (r = -281.3). Therefore, the sum of roots (a + b + c = 20), the sum of products (ab + bc + ac = 131), and the product (abc = 281.3). Okay, so semi-perimeter (s) is (frac{a + b + c}{2} = frac{20}{2} = 10). That's straightforward. Now, Heron's formula squared is (s(s - a)(s - b)(s - c)). So we need to compute that. But how can we compute ( (s - a)(s - b)(s - c) ) without knowing the individual values of (a), (b), and (c)?Let me think. Let's denote (s = 10). Then ( (10 - a)(10 - b)(10 - c) ). Expanding this expression might help. Let's try expanding it. First, ( (10 - a)(10 - b) = 100 - 10a - 10b + ab ). Then multiplying by ( (10 - c) ):( [100 - 10a - 10b + ab] times (10 - c) )Multiply term by term:= 100*10 - 100c -10a*10 + 10a c -10b*10 + 10b c + ab*10 - ab cSimplify each term:= 1000 - 100c - 100a + 10ac - 100b + 10bc + 10ab - abcCombine like terms:= 1000 - 100(a + b + c) + 10(ac + bc + ab) - abcNow substitute the known values from Vieta's formulas:We know (a + b + c = 20), (ab + bc + ac = 131), (abc = 281.3). Plugging these in:= 1000 - 100*20 + 10*131 - 281.3Calculate each term:First, 100*20 = 200010*131 = 1310So:= 1000 - 2000 + 1310 - 281.3Now compute step by step:1000 - 2000 = -1000-1000 + 1310 = 310310 - 281.3 = 28.7Therefore, ( (10 - a)(10 - b)(10 - c) = 28.7 )Then, Heron's formula squared is ( s times 28.7 = 10 times 28.7 = 287 )Wait, so the square of the area is 287? Hmm. Let me check my calculations again to be sure.First, expanding ( (10 - a)(10 - b)(10 - c) ):Let me write the expansion again in terms of symmetric sums:( (s - a)(s - b)(s - c) = s^3 - s^2(a + b + c) + s(ab + bc + ac) - abc )Wait, this is a standard formula for expanding such products. Let me confirm. For any cubic equation with roots (a, b, c), the polynomial can be written as ( (x - a)(x - b)(x - c) = x^3 - (a + b + c)x^2 + (ab + bc + ac)x - abc ). Therefore, if we substitute (x = s), we get exactly the expansion above. So yes, ( (s - a)(s - b)(s - c) = s^3 - s^2(a + b + c) + s(ab + bc + ac) - abc ).So substituting (s = 10), and the values:= 10^3 - 10^2*(20) + 10*(131) - 281.3Compute each term:10^3 = 100010^2*20 = 100*20 = 200010*131 = 1310So:1000 - 2000 + 1310 - 281.3Yes, same as before:1000 - 2000 = -1000-1000 + 1310 = 310310 - 281.3 = 28.7Therefore, ( (10 - a)(10 - b)(10 - c) = 28.7 )Multiply by (s = 10) gives the square of the area: 10 * 28.7 = 287So the answer is 287. Therefore, the square of the area is (boxed{287}). Wait, but the problem states the polynomial is (x^3 -20x^2 +131x -281.3). The decimal at the end, 281.3, is that exact? Hmm, 281.3 is equivalent to 2813/10. Maybe the problem expects an exact fractional form? Let me check.But since the answer is 287, which is an integer, and 28.7 multiplied by 10 is 287, perhaps 287 is correct. Alternatively, maybe the decimal is a red herring, but since we followed through with the calculation, and all steps check out, I think 287 is the right answer.But let me just verify once more. Let's recast the problem.Given a cubic polynomial with roots a, b, c (sides of the triangle), sum a + b + c = 20, ab + bc + ac = 131, abc = 281.3. Then Heron's formula squared is s(s - a)(s - b)(s - c). s = 10. Then s - a = 10 - a, etc. So expanding (10 - a)(10 - b)(10 - c) gives 1000 - 2000 + 1310 - 281.3 = 28.7. Multiply by 10 gives 287. Seems correct.Alternatively, is there a mistake in the sign? Let's check the expansion again. The formula is (s - a)(s - b)(s - c) = s^3 - s^2(a + b + c) + s(ab + bc + ac) - abc. So with s = 10, a + b + c = 20, ab + bc + ac = 131, abc = 281.3:10^3 - 10^2*20 + 10*131 - 281.3 = 1000 - 2000 + 1310 - 281.31000 - 2000 is -1000. -1000 + 1310 is 310. 310 - 281.3 is 28.7. So yes, that's correct. Then multiplying by s (which is 10) gives 287. So the square of the area is 287. Therefore, the answer is 287. Seems correct.I don't see any errors in the reasoning. So I think the answer is boxed{287}.Final Answerboxed{287}</think>Given the polynomial (x^3 - 20x^2 + 131x - 281.3) with roots (a), (b), and (c) which are the side lengths of a scalene triangle, we need to find the square of the area of the triangle.Using Vieta's formulas, we know:- The sum of the roots: (a + b + c = 20)- The sum of the products of the roots taken two at a time: (ab + bc + ac = 131)- The product of the roots: (abc = 281.3)The semi-perimeter (s) of the triangle is:[s = frac{a + b + c}{2} = frac{20}{2} = 10]Using Heron's formula, the square of the area of the triangle is given by:[text{Area}^2 = s(s - a)(s - b)(s - c)]We need to expand ((s - a)(s - b)(s - c)). Using the polynomial expansion for (x = s):[(s - a)(s - b)(s - c) = s^3 - s^2(a + b + c) + s(ab + bc + ac) - abc]Substituting the known values:[(10 - a)(10 - b)(10 - c) = 10^3 - 10^2 cdot 20 + 10 cdot 131 - 281.3]Calculating each term:[10^3 = 1000][10^2 cdot 20 = 2000][10 cdot 131 = 1310]Combining these:[1000 - 2000 + 1310 - 281.3 = -1000 + 1310 - 281.3 = 310 - 281.3 = 28.7]Thus, ((10 - a)(10 - b)(10 - c) = 28.7). Multiplying by the semi-perimeter (s = 10):[text{Area}^2 = 10 cdot 28.7 = 287]The square of the area of the triangle is (boxed{287}).]

🔑:1. Let the side lengths of the scalene triangle be (a), (b), and (c). These are the roots of the polynomial (x^3 - 20x^2 + 131x - 281.3).2. By Vieta's formulas, we know: [ a + b + c = 20, ] [ ab + bc + ca = 131, ] [ abc = 281.3. ]3. To find the square of the area of the triangle, we use Heron's formula. First, we compute the semi-perimeter (s): [ s = frac{a + b + c}{2} = frac{20}{2} = 10. ]4. Heron's formula for the area (A) of a triangle is: [ A = sqrt{s(s-a)(s-b)(s-c)}. ] We need to find (s(s-a)(s-b)(s-c)): [ s(s-a)(s-b)(s-c) = 10 cdot (10-a) cdot (10-b) cdot (10-c). ]5. We expand ((10-a)(10-b)(10-c)): [ (10-a)(10-b)(10-c) = 1000 - 100(a+b+c) + 10(ab+bc+ca) - abc. ]6. Substitute the values from Vieta's formulas: [ (10-a)(10-b)(10-c) = 1000 - 100 cdot 20 + 10 cdot 131 - 281.3. ]7. Simplify the expression: [ (10-a)(10-b)(10-c) = 1000 - 2000 + 1310 - 281.3 = 28.7. ]8. Therefore: [ s(s-a)(s-b)(s-c) = 10 cdot 28.7 = 287. ]9. The square of the area of the triangle is: [ A^2 = s(s-a)(s-b)(s-c) = 287. ]The final answer is (boxed{287}).

❓:9. Let the function f(x)=5(x+1)^{2}+frac{a}{(x+1)^{5}}(a>0), when x geqslant 0, f(x) geqslant 24, the minimum value of a is

🧠:[<think>Okay, so I need to find the minimum value of ( a ) such that the function ( f(x) = 5(x+1)^2 + frac{a}{(x+1)^5} ) is always greater than or equal to 24 for ( x geq 0 ). The problem states that ( a > 0 ). Alright, let's start by understanding the problem.First, the function is defined for ( x geq 0 ), so ( x + 1 geq 1 ), which means the denominators ( (x+1)^2 ) and ( (x+1)^5 ) are all positive. That's good because we don't have to worry about division by zero or negative denominators here. Since ( a ) is positive, both terms in the function ( f(x) ) are positive. The question is asking for the smallest ( a ) such that ( f(x) ) never drops below 24 for any ( x geq 0 ).Hmm, so we need to ensure that the minimum value of ( f(x) ) over ( x geq 0 ) is at least 24. Therefore, the strategy here is to find the minimum of ( f(x) ) in the domain ( x geq 0 ), set that minimum equal to 24, and solve for ( a ). The smallest such ( a ) that satisfies this condition would be our answer.To find the minimum of ( f(x) ), I should take the derivative of ( f(x) ) with respect to ( x ), set it equal to zero, and solve for ( x ). Then, check if that critical point is a minimum by using the second derivative test or analyzing the behavior of the first derivative. Once the critical point is found, substitute it back into ( f(x) ) and set the result equal to 24 to solve for ( a ).Alright, let's compute the first derivative ( f'(x) ).Given:[f(x) = 5(x + 1)^2 + frac{a}{(x + 1)^5}]Let's rewrite ( f(x) ) to make differentiation easier:[f(x) = 5(x + 1)^2 + a(x + 1)^{-5}]Now, differentiate term by term.First term derivative:[frac{d}{dx} [5(x + 1)^2] = 5 times 2(x + 1)^{1} = 10(x + 1)]Second term derivative:[frac{d}{dx} [a(x + 1)^{-5}] = a times (-5)(x + 1)^{-6} times 1 = -5a(x + 1)^{-6}]So, combining these, the first derivative is:[f'(x) = 10(x + 1) - frac{5a}{(x + 1)^6}]To find critical points, set ( f'(x) = 0 ):[10(x + 1) - frac{5a}{(x + 1)^6} = 0]Let's solve for ( x ). First, move the second term to the other side:[10(x + 1) = frac{5a}{(x + 1)^6}]Multiply both sides by ( (x + 1)^6 ):[10(x + 1)^7 = 5a]Divide both sides by 5:[2(x + 1)^7 = a]So, ( a = 2(x + 1)^7 ).This equation relates ( a ) and the critical point ( x ). Since we are looking for the minimal ( a ), we need to find the ( x ) that minimizes ( a ). Wait, but here ( a ) is expressed in terms of ( x ). So, if we can express ( a ) in terms of ( x ), then perhaps we can find the minimal ( a ) such that ( f(x) geq 24 ).But actually, the critical point gives a relation between ( a ) and ( x ). For each ( a ), there's a critical point at ( x ) such that ( a = 2(x + 1)^7 ). But since we need the minimal ( a ) such that the function never goes below 24, we need to ensure that at the critical point (which is the minimum point of the function), the value of ( f(x) ) is exactly 24.Wait, perhaps a better approach is to note that for the function to have a minimum of 24, the minimal value occurs at the critical point, so we can substitute ( a = 2(x + 1)^7 ) into the original function ( f(x) ), set it equal to 24, and solve for ( x ). Then, from that ( x ), compute ( a ).Let me test this idea.So, substitute ( a = 2(x + 1)^7 ) into ( f(x) ):[f(x) = 5(x + 1)^2 + frac{2(x + 1)^7}{(x + 1)^5}]Simplify the second term:[frac{2(x + 1)^7}{(x + 1)^5} = 2(x + 1)^{7 - 5} = 2(x + 1)^2]Therefore:[f(x) = 5(x + 1)^2 + 2(x + 1)^2 = 7(x + 1)^2]Set this equal to 24:[7(x + 1)^2 = 24]Solve for ( x + 1 ):[(x + 1)^2 = frac{24}{7}]Take square roots:[x + 1 = sqrt{frac{24}{7}} = sqrt{frac{24}{7}} = frac{2sqrt{42}}{7}]Wait, let's compute ( sqrt{24/7} ):[sqrt{frac{24}{7}} = sqrt{frac{24}{7}} = frac{sqrt{24}}{sqrt{7}} = frac{2sqrt{6}}{sqrt{7}} = frac{2sqrt{42}}{7}]Yes, because ( sqrt{6}/sqrt{7} = sqrt{42}/7 ).Therefore, ( x + 1 = frac{2sqrt{42}}{7} ), so ( x = frac{2sqrt{42}}{7} - 1 ).But wait, since ( x geq 0 ), we need to check if this ( x ) is in the domain. Let's compute ( frac{2sqrt{42}}{7} ):First, ( sqrt{42} approx 6.4807 ), so ( 2 times 6.4807 approx 12.9614 ), divided by 7 is approximately 1.8516. So, ( x approx 1.8516 - 1 = 0.8516 ), which is positive. So, ( x approx 0.8516 ) is within the domain ( x geq 0 ). Good.Therefore, at this critical point, the value of ( f(x) ) is 24, given ( a = 2(x + 1)^7 ). So, substituting ( x + 1 = frac{2sqrt{42}}{7} ) into ( a = 2(x + 1)^7 ):Compute ( a ):First, compute ( (x + 1)^7 = left( frac{2sqrt{42}}{7} right)^7 )But this seems complicated. Wait, maybe there's a smarter way here.Wait, let's recall that when we set ( f'(x) = 0 ), we found ( a = 2(x + 1)^7 ). Then substituting back into ( f(x) ), we found ( f(x) = 7(x + 1)^2 = 24 ), so ( (x + 1)^2 = 24/7 ), so ( (x + 1)^7 = (24/7)^{7/2} ). Therefore, ( a = 2 times (24/7)^{7/2} ).But that seems complex, perhaps we can compute it more straightforwardly.Alternatively, since we have ( (x + 1)^2 = 24/7 ), then ( (x + 1)^7 = (x + 1)^2 times (x + 1)^5 = (24/7) times (x + 1)^5 ). But from the original function, the second term is ( a/(x + 1)^5 ). Wait, let's see.From the original function, we had:( f(x) = 5(x + 1)^2 + frac{a}{(x + 1)^5} = 24 )But we also found that at the critical point, ( a = 2(x + 1)^7 ). So substituting back, the second term becomes ( 2(x + 1)^7 / (x + 1)^5 = 2(x + 1)^2 ). Then the function is ( 5(x + 1)^2 + 2(x + 1)^2 = 7(x + 1)^2 = 24 ), which gives ( (x + 1)^2 = 24/7 ), so ( (x + 1) = sqrt{24/7} ).Therefore, ( a = 2 times (sqrt{24/7})^7 ).Compute ( (sqrt{24/7})^7 ):First, note that ( (sqrt{24/7})^7 = (24/7)^{7/2} = (24/7)^{3 + 1/2} = (24/7)^3 times sqrt{24/7} ).Compute ( (24/7)^3 ):24^3 = 24*24*24 = 576*24 = 13,8247^3 = 343So, (24/7)^3 = 13,824 / 343 ≈ 40.29 (exact value is 13,824 ÷ 343 ≈ 40.29)Then, multiply by ( sqrt{24/7} approx sqrt{3.4286} ≈ 1.8516 )So, approximately, 40.29 * 1.8516 ≈ 74.64Then, ( a = 2 * 74.64 ≈ 149.28 )But this is an approximate value. Let's try to compute it exactly.Alternatively, we can express ( a ) in terms of exponents.Given that ( a = 2 times (24/7)^{7/2} )Express 24 as 8*3, so 24/7 = 8*3 /7Therefore, ( (24/7)^{7/2} = (8*3/7)^{7/2} = 8^{7/2} * 3^{7/2} / 7^{7/2} )But 8^{7/2} = (2^3)^{7/2} = 2^{21/2} = 2^{10 + 1/2} = 2^{10} * sqrt(2) = 1024 * 1.4142 ≈ 1448.323^{7/2} = 3^3 * sqrt(3) = 27 * 1.732 ≈ 46.7647^{7/2} = 7^3 * sqrt(7) = 343 * 2.6458 ≈ 907.35Multiply numerator: 1448.32 * 46.764 ≈ 1448.32 * 40 = 57,932.8; 1448.32 * 6.764 ≈ ~9,800. Total ≈ 67,732.8Divide by denominator ≈ 907.35:67,732.8 / 907.35 ≈ 74.66Then multiply by 2: 74.66 * 2 ≈ 149.32So, approximate value is around 149.32.But the problem says "the minimum value of ( a )", and it's probably expecting an exact expression. Let's see.Given ( a = 2 times (24/7)^{7/2} )But perhaps we can write it as ( 2 times (24)^{7/2} / 7^{7/2} )But 24 = 4*6, so 24^{7/2} = (4*6)^{7/2} = 4^{7/2} * 6^{7/2} = (2^2)^{7/2} * 6^{7/2} = 2^7 * 6^{7/2} = 128 * (6^{3} * sqrt(6)) ) = 128 * 216 * sqrt(6) = 27,648 * sqrt(6)Similarly, 7^{7/2} = 7^3 * sqrt(7) = 343 * sqrt(7)Therefore, ( a = 2 * (27,648 * sqrt(6)) / (343 * sqrt(7)) ) = (2 * 27,648 / 343) * (sqrt(6)/sqrt(7)) )Compute 2*27,648 = 55,29655,296 / 343 ≈ let's divide 55,296 by 343:343 * 160 = 54,88055,296 - 54,880 = 416416 / 343 ≈ 1.213So, approximately 161.213Then, sqrt(6)/sqrt(7) = sqrt(42)/7 ≈ 6.4807/7 ≈ 0.9258Therefore, total a ≈ 161.213 * 0.9258 ≈ 149.32, which matches our previous approximation.But perhaps there's a better way to write this. Let's see:( a = 2 times left( frac{24}{7} right)^{7/2} )Alternatively, since ( 24 = 4 times 6 ), we can write:( a = 2 times left( frac{4 times 6}{7} right)^{7/2} = 2 times left( frac{4}{7} times 6 right)^{7/2} = 2 times left( frac{4}{7} right)^{7/2} times 6^{7/2} )But this might not be helpful. Alternatively, express in terms of exponents:( a = 2 times 24^{7/2} / 7^{7/2} )But this is still complicated.Wait, maybe the problem expects an exact form, so leaving it as ( a = 2 times left( frac{24}{7} right)^{7/2} ), but that might not be simplified correctly. Let me check.Alternatively, note that when we had:From ( 7(x + 1)^2 = 24 ), so ( (x + 1)^2 = 24/7 ), so ( (x + 1)^7 = (x + 1)^2 times (x + 1)^5 = (24/7) times (x + 1)^5 ).But in the original function, ( f(x) = 5(x + 1)^2 + frac{a}{(x + 1)^5} ). At the critical point, we have ( a = 2(x + 1)^7 ), so substituting back into the original function, we have:( 5(x + 1)^2 + frac{2(x + 1)^7}{(x + 1)^5} = 5(x + 1)^2 + 2(x + 1)^2 = 7(x + 1)^2 = 24 ), so ( (x + 1)^2 = 24/7 ), hence ( (x + 1)^7 = (24/7)^{7/2} ), so ( a = 2 times (24/7)^{7/2} ).Alternatively, note that ( (24/7)^{7/2} = 24^{7/2} / 7^{7/2} ). However, simplifying this further is perhaps not necessary. Therefore, the exact value of ( a ) is ( 2 times (24/7)^{7/2} ). However, we can write this in terms of radicals:( a = 2 times left( sqrt{frac{24}{7}} right)^7 = 2 times left( frac{2sqrt{42}}{7} right)^7 )Wait, earlier we had ( x + 1 = sqrt{24/7} ), which we simplified as ( sqrt{24}/sqrt{7} = 2sqrt{6}/sqrt{7} = 2sqrt{42}/7 ). Therefore, ( sqrt{24/7} = 2sqrt{42}/7 ).Therefore, ( a = 2 times left( 2sqrt{42}/7 right)^7 ).Let's compute this:First, write ( 2sqrt{42}/7 ) as ( (2/7)sqrt{42} ).Therefore, ( a = 2 times left( (2/7)sqrt{42} right)^7 = 2 times (2/7)^7 times (sqrt{42})^7 )Compute ( (sqrt{42})^7 = (42)^{7/2} = 42^{3} times sqrt{42} = 74088 times sqrt{42} )Therefore, ( a = 2 times (2^7 / 7^7) times 74088 times sqrt{42} )Compute 2^7 = 128, 7^7 = 823543So, ( a = 2 times (128 / 823543) times 74088 times sqrt{42} )Calculate 128/823543 ≈ 0.00015540.0001554 * 74088 ≈ 0.0001554 * 70,000 ≈ 10.878; 0.0001554 * 4,088 ≈ 0.0001554 * 4,000 ≈ 0.6216; total ≈ 11.4996Multiply by 2: ≈ 22.9992So, ( a ≈ 22.9992 times sqrt{42} )But ( sqrt(42) ≈ 6.4807 ), so 22.9992 * 6.4807 ≈ 149.3, which matches our earlier approximation.But this seems like a very convoluted way to compute it, and perhaps the problem expects an exact form, which would be ( 2 times (24/7)^{7/2} ). However, maybe there's a smarter substitution or a way to apply the AM-GM inequality here since the function is a sum of two terms, which might be amenable to that.Wait, AM-GM inequality. Let me think.The function ( f(x) = 5(x+1)^2 + frac{a}{(x+1)^5} ). If we can express this as a sum of terms that can be handled by AM-GM, that might lead us to the minimal ( a ).But the standard AM-GM inequality states that for non-negative real numbers, the arithmetic mean is at least the geometric mean. So, if we can write the sum of terms such that their product is a constant, we can apply AM-GM.However, here we have two terms: ( 5(x + 1)^2 ) and ( frac{a}{(x + 1)^5} ). Let's see if we can split the first term into multiple terms so that all terms can be multiplied to get a constant.Alternatively, let's think of variable substitution. Let ( t = x + 1 ), so ( t geq 1 ) (since ( x geq 0 )). Then, the function becomes:( f(t) = 5t^2 + frac{a}{t^5} )We need ( f(t) geq 24 ) for all ( t geq 1 ). We need to find the minimal ( a > 0 ) such that this holds.So, the problem reduces to: find the minimal ( a > 0 ) such that ( 5t^2 + frac{a}{t^5} geq 24 ) for all ( t geq 1 ).To find the minimal ( a ), we need to ensure that the minimum of ( 5t^2 + frac{a}{t^5} ) over ( t geq 1 ) is exactly 24. Therefore, the minimal ( a ) occurs when the function touches the line ( y = 24 ) at its minimum point. So, similar to before, find the t where the minimum occurs, set the function equal to 24 there, and solve for ( a ).Alternatively, using calculus, take derivative of ( f(t) ):( f'(t) = 10t - frac{5a}{t^6} )Set to zero:( 10t = frac{5a}{t^6} )Multiply both sides by ( t^6 ):( 10t^7 = 5a )Simplify:( 2t^7 = a )Therefore, at critical point, ( a = 2t^7 ). Then, substitute back into ( f(t) ):( f(t) = 5t^2 + frac{2t^7}{t^5} = 5t^2 + 2t^2 = 7t^2 )Set equal to 24:( 7t^2 = 24 Rightarrow t^2 = frac{24}{7} Rightarrow t = sqrt{frac{24}{7}} )Therefore, ( a = 2t^7 = 2 left( sqrt{frac{24}{7}} right)^7 )Which is the same as before. So, exact value is ( 2 left( frac{24}{7} right)^{7/2} ). Alternatively, since ( t geq 1 ), check if ( sqrt{24/7} geq 1 ). Since 24/7 ≈ 3.4286, sqrt of that ≈ 1.8516 ≥ 1, so yes, it's within the domain.Therefore, the minimal ( a ) is ( 2 times (24/7)^{7/2} ). However, this is an exact expression but perhaps can be simplified more.Alternatively, note that ( (24)^{7/2} = 24^{3} times sqrt{24} = 13,824 times 2sqrt{6} = 27,648 sqrt{6} ). Then, ( (7)^{7/2} = 7^3 times sqrt{7} = 343 sqrt{7} ). Therefore,( a = 2 times frac{27,648 sqrt{6}}{343 sqrt{7}} = 2 times frac{27,648}{343} times sqrt{frac{6}{7}} )Simplify ( sqrt{frac{6}{7}} = frac{sqrt{42}}{7} )So,( a = 2 times frac{27,648}{343} times frac{sqrt{42}}{7} = 2 times frac{27,648}{343 times 7} times sqrt{42} )Calculate ( 343 times 7 = 2,401 )So,( a = 2 times frac{27,648}{2,401} times sqrt{42} )Compute ( 27,648 ÷ 2,401 ):2,401 * 11 = 26,41127,648 - 26,411 = 1,2372,401 * 0.5 ≈ 1,200.5So, 1,237 ÷ 2,401 ≈ 0.515Therefore, approximately 11.515Multiply by 2: 23.03Multiply by ( sqrt{42} ≈ 6.4807 ): 23.03 * 6.4807 ≈ 149.3Again, the same approximate value. Therefore, the exact value is ( 2 times (24/7)^{7/2} ), and the approximate decimal is about 149.3. However, the problem might expect the answer in terms of radicals or exponents. Alternatively, maybe there's a calculation error here. Wait, let's check the initial substitution again.Wait, when we substituted ( a = 2t^7 ) into the original equation, we had:( 5t^2 + frac{a}{t^5} = 5t^2 + frac{2t^7}{t^5} = 5t^2 + 2t^2 = 7t^2 = 24 ), so ( t^2 = 24/7 ), so ( t = sqrt{24/7} ). Then, ( a = 2t^7 = 2 times (24/7)^{7/2} ). So, yes, that's exact. Therefore, the minimal value of ( a ) is ( 2 times (24/7)^{7/2} ).But let's check if this can be written in a simpler form. For example, note that ( (24/7)^{7/2} = 24^{3} times sqrt{24/7} ). Then, ( 24^3 = 13,824 ), so:( a = 2 times 13,824 times sqrt{24/7} div 7^{3} )Wait, ( 24^{7/2} = 24^{3 + 1/2} = 24^3 times 24^{1/2} = 13,824 times sqrt{24} ). Similarly, ( 7^{7/2} = 7^3 times 7^{1/2} = 343 times sqrt{7} ).Therefore, ( a = 2 times (24^{7/2}/7^{7/2}) = 2 times (13,824 times sqrt{24}) / (343 times sqrt{7}) ) )Simplify ( sqrt{24} = 2sqrt{6} ), so:( a = 2 times (13,824 times 2sqrt{6}) / (343 times sqrt{7}) ) = 2 times (27,648 sqrt{6}) / (343 sqrt{7}) )Rationalize the denominator:Multiply numerator and denominator by ( sqrt{7} ):( a = 2 times (27,648 sqrt{6} sqrt{7}) / (343 times 7) ) = 2 times (27,648 sqrt{42}) / (2,401) )Therefore,( a = (55,296 sqrt{42}) / 2,401 )Simplify 55,296 ÷ 2,401:Divide numerator and denominator by 7:55,296 ÷7 = 7,899.4282,401 ÷7 = 343So, 7,899.428 / 343 ≈ 23.03Thus, ( a ≈ 23.03 sqrt{42} approx 23.03 * 6.4807 ≈ 149.3 ). Same as before.Therefore, the exact value is ( a = frac{55,296 sqrt{42}}{2,401} ). But this can be simplified further by dividing numerator and denominator by GCD(55,296, 2,401). Let's check:Find GCD of 55,296 and 2,401.Divide 55,296 by 2,401:2,401 * 23 = 55,22355,296 - 55,223 = 73So, GCD(2,401, 73). Check if 73 divides 2,401:73 * 32 = 2,336; 2,401 - 2,336 = 65GCD(73,65) = GCD(65,8) = GCD(8,1) = 1.Therefore, GCD is 1. So, the fraction cannot be simplified further.Therefore, the exact value is ( frac{55,296 sqrt{42}}{2,401} ), but this is likely not the expected answer. Alternatively, the answer is ( 2 times (24/7)^{7/2} ), which can be written as ( 2 times left( frac{24}{7} right)^{3.5} ).However, given that the problem is likely expecting an exact answer, and given the options, perhaps expressed in terms of radicals, the answer is ( 2 times left( sqrt{frac{24}{7}} right)^7 ), which is equivalent.But maybe there's an error in my approach. Let me verify.Wait, perhaps I can use the AM-GM inequality here. Let's try.The function is ( f(t) = 5t^2 + frac{a}{t^5} ), ( t geq 1 ).We need to ensure ( 5t^2 + frac{a}{t^5} geq 24 ).To apply AM-GM, we need to write the expression as a sum of positive terms whose product is a constant. Let's see.Suppose we split the first term into multiple terms. Let's split 5t^2 into five terms of t^2 each. Then, we have:( f(t) = t^2 + t^2 + t^2 + t^2 + t^2 + frac{a}{t^5} )Now, we have six terms: five times t^2 and one term of ( frac{a}{t^5} ).Applying AM-GM on these six terms:The arithmetic mean is ( frac{5t^2 + frac{a}{t^5}}{6} geq sqrt[6]{t^2 times t^2 times t^2 times t^2 times t^2 times frac{a}{t^5}} )Compute the geometric mean:Inside the root:( sqrt[6]{t^{10} times frac{a}{t^5}} = sqrt[6]{a t^{5}} )Therefore,( frac{5t^2 + frac{a}{t^5}}{6} geq sqrt[6]{a t^{5}} )Multiply both sides by 6:( 5t^2 + frac{a}{t^5} geq 6 sqrt[6]{a t^{5}} )But we need ( 5t^2 + frac{a}{t^5} geq 24 ). To make the inequality hold via AM-GM, we need the right-hand side to be at least 24. So:( 6 sqrt[6]{a t^{5}} geq 24 )Divide both sides by 6:( sqrt[6]{a t^{5}} geq 4 )Raise both sides to the 6th power:( a t^{5} geq 4^6 = 4096 )Therefore,( a geq frac{4096}{t^5} )But this must hold for all ( t geq 1 ). However, as ( t ) increases, ( frac{4096}{t^5} ) decreases. Therefore, the maximum of ( frac{4096}{t^5} ) occurs at ( t = 1 ), giving ( a geq 4096 ). But this contradicts our previous result of approximately 149.3. Therefore, AM-GM in this way gives a much larger lower bound for ( a ), which is not useful. Therefore, this approach might not be helpful here.Alternatively, perhaps we didn't split the terms correctly. Let's think differently.If we consider the two terms ( 5t^2 ) and ( frac{a}{t^5} ), perhaps we can use weighted AM-GM. But I'm not sure. Alternatively, perhaps consider writing the expression as ( 5t^2 + frac{a}{t^5} = k ), and find the minimal k, but that's similar to what we did with calculus.Alternatively, think of the function ( f(t) = 5t^2 + frac{a}{t^5} ) and set its minimum to 24. We already did this with calculus, and the answer is ( a = 2 times (24/7)^{7/2} approx 149.3 ). Therefore, given that the problem states ( a > 0 ), and asks for the minimal such ( a ), the answer should be ( boxed{2 times (24/7)^{7/2}} ). However, maybe the answer is expected to be simplified in another form or perhaps as a whole number. But 149.3 is approximately 149.3, but the exact value is an irrational number.Wait, but when I compute ( 2 times (24/7)^{7/2} ), let's compute this step by step:First, compute ( (24/7)^{1/2} = sqrt{24/7} ≈ 1.8516 )Then, ( (24/7)^{7/2} = (24/7)^3 times (24/7)^{1/2} approx (13.824/343) * 1.8516 ≈ (13.824/343 is about 0.0403) * 1.8516 ≈ 0.0746. Wait, that can't be right. Wait, no:Wait, ( (24/7)^{3} = (24^3)/(7^3) = 13,824 / 343 ≈ 40.299( (24/7)^{1/2} ≈ 1.8516 )Therefore, ( (24/7)^{7/2} = 40.299 * 1.8516 ≈ 74.66Then, ( 2 * 74.66 ≈ 149.32 ). So, the exact value is approximately 149.32. However, since the problem says "the minimum value of ( a )", and given that ( a ) is positive, the minimal ( a ) is this exact value. However, the problem might be expecting an exact form in terms of a fraction with exponents or something else, but I think the answer is supposed to be written as ( boxed{2 left( frac{24}{7} right)^{7/2}} ). However, maybe there's a miscalculation here.Wait, let me verify the calculus approach again. Maybe I made a mistake in substituting back.Given ( f(x) = 5(x + 1)^2 + frac{a}{(x + 1)^5} )Take derivative: ( f'(x) = 10(x + 1) - 5a/(x + 1)^6 )Set to zero: ( 10(x + 1) = 5a/(x + 1)^6 )Multiply both sides by ( (x + 1)^6 ): ( 10(x + 1)^7 = 5a Rightarrow 2(x + 1)^7 = a )So, ( a = 2(x + 1)^7 ). Then, substitute back into f(x):( f(x) = 5(x + 1)^2 + 2(x + 1)^7/(x + 1)^5 = 5(x + 1)^2 + 2(x + 1)^2 = 7(x + 1)^2 )Set equal to 24: ( 7(x + 1)^2 = 24 Rightarrow (x + 1)^2 = 24/7 Rightarrow x + 1 = sqrt{24/7} )Thus, ( a = 2*(sqrt(24/7))^7 = 2*(24/7)^{7/2} ), which is correct.Therefore, the minimal ( a ) is ( 2*(24/7)^{7/2} ). This is the exact value, and the approximate is roughly 149.3. If the problem expects an exact answer, this is it. If it expects a decimal, then approximately 149.3, but given that mathematical problems often prefer exact forms, especially since 24 and 7 are integers, and the exponent is a fraction, I think the answer should be presented as ( boxed{2 left( dfrac{24}{7} right)^{7/2}} ).However, let me check if there's another approach or if the answer can be expressed differently. Wait, perhaps rational exponents:( left( frac{24}{7} right)^{7/2} = left( frac{24}{7} right)^3 times left( frac{24}{7} right)^{1/2} = frac{24^3}{7^3} times frac{sqrt{24}}{sqrt{7}} = frac{13,824}{343} times frac{2sqrt{6}}{sqrt{7}} = frac{13,824 times 2 sqrt{6}}{343 sqrt{7}} = frac{27,648 sqrt{42}}{343 times 7} = frac{27,648 sqrt{42}}{2,401} )Then, ( a = 2 times frac{27,648 sqrt{42}}{2,401} = frac{55,296 sqrt{42}}{2,401} ), but this is the same as before.Alternatively, since 55,296 divided by 2,401 is approximately 23.03, as computed earlier, so this is ( 23.03 sqrt{42} ), but unless there's a simplification, this is as far as we can go.Therefore, the answer is ( boxed{2 left( dfrac{24}{7} right)^{7/2}} ).Final AnswerThe minimum value of ( a ) is boxed{2 left( dfrac{24}{7} right)^{7/2}}.</think>To find the minimum value of ( a ) such that the function ( f(x) = 5(x+1)^2 + frac{a}{(x+1)^5} ) is always greater than or equal to 24 for ( x geq 0 ), we need to ensure that the minimum value of ( f(x) ) is at least 24. First, we find the critical points by taking the derivative of ( f(x) ):[f'(x) = 10(x + 1) - frac{5a}{(x + 1)^6}]Setting the derivative equal to zero to find critical points:[10(x + 1) = frac{5a}{(x + 1)^6}]Solving for ( a ):[2(x + 1)^7 = a]Substituting ( a = 2(x + 1)^7 ) back into the original function ( f(x) ):[f(x) = 5(x + 1)^2 + frac{2(x + 1)^7}{(x + 1)^5} = 5(x + 1)^2 + 2(x + 1)^2 = 7(x + 1)^2]Setting this equal to 24:[7(x + 1)^2 = 24 implies (x + 1)^2 = frac{24}{7} implies x + 1 = sqrt{frac{24}{7}}]Substituting ( x + 1 = sqrt{frac{24}{7}} ) into the expression for ( a ):[a = 2 left( sqrt{frac{24}{7}} right)^7 = 2 left( frac{24}{7} right)^{7/2}]Thus, the minimum value of ( a ) is:[boxed{2 left( dfrac{24}{7} right)^{7/2}}],<think>Okay, let me try to figure out this problem. So, we have the function f(x) = 5(x + 1)^2 + a/(x + 1)^5, where a is greater than 0. We need to find the minimum value of a such that f(x) is always greater than or equal to 24 for all x greater than or equal to 0. Hmm, okay. First, I remember that when dealing with functions like this where you have a variable in both the numerator and denominator, especially with exponents, calculus might be useful here. Specifically, finding the minimum value of the function f(x) on the interval x ≥ 0, and then setting that minimum equal to 24. Then solve for a. Since a is a positive constant, we need to find the smallest a that ensures the function never dips below 24. So, maybe I should start by finding the critical points of f(x). Critical points occur where the derivative is zero or undefined. Let's compute the derivative f'(x). Given f(x) = 5(x + 1)^2 + a/(x + 1)^5. Let's rewrite that as 5(x + 1)^2 + a(x + 1)^(-5). Then the derivative f'(x) would be:First term derivative: 5 * 2(x + 1) = 10(x + 1)Second term derivative: a * (-5)(x + 1)^(-6) = -5a(x + 1)^(-6)So overall, f'(x) = 10(x + 1) - 5a/(x + 1)^6To find critical points, set f'(x) = 0:10(x + 1) - 5a/(x + 1)^6 = 0Let me factor out 5/(x + 1)^6 to make it easier. Wait, maybe not. Let me rearrange the equation:10(x + 1) = 5a/(x + 1)^6Multiply both sides by (x + 1)^6:10(x + 1)^7 = 5aDivide both sides by 5:2(x + 1)^7 = aSo, a = 2(x + 1)^7Hmm, interesting. So this gives a relationship between a and x at the critical points. Since we are looking for the minimum value of a such that f(x) >= 24 for all x >= 0, perhaps the minimum occurs when a is as small as possible to satisfy f(x) = 24 at its critical point. So, we need to find the x where the function has a minimum (critical point) and set that minimum equal to 24, then solve for a.But since a is expressed in terms of x here (a = 2(x + 1)^7), we can substitute that back into the original function f(x) to find the value of the function at the critical point.So, substitute a = 2(x + 1)^7 into f(x):f(x) = 5(x + 1)^2 + [2(x + 1)^7]/(x + 1)^5Simplify the second term: [2(x + 1)^7]/ (x + 1)^5 = 2(x + 1)^(7 - 5) = 2(x + 1)^2Therefore, f(x) becomes:5(x + 1)^2 + 2(x + 1)^2 = 7(x + 1)^2So, at the critical point, f(x) = 7(x + 1)^2. But we need this critical point to be equal to 24. Therefore:7(x + 1)^2 = 24Solve for (x + 1)^2:(x + 1)^2 = 24/7Take square roots:x + 1 = sqrt(24/7) = (2*sqrt(42))/7 ≈ but maybe leave it in exact form.Wait, 24/7 is under the square root. Let's see sqrt(24/7) can be simplified as sqrt(24)/sqrt(7) = 2*sqrt(6)/sqrt(7) = 2*sqrt(42)/7. Hmm, perhaps. But maybe we can just leave it as sqrt(24/7).But then, if x + 1 = sqrt(24/7), then x = sqrt(24/7) - 1. But x must be greater than or equal to 0, so we need to check if sqrt(24/7) - 1 is non-negative.Compute sqrt(24/7): sqrt(24)/sqrt(7) ≈ 4.89898/2.64575 ≈ 1.8516. Then subtract 1, so x ≈ 0.8516, which is positive. So that's valid.But now, we can relate back to a. Remember, at this critical point, a = 2(x + 1)^7.So substitute x + 1 = sqrt(24/7):a = 2*(sqrt(24/7))^7But sqrt(24/7) is (24/7)^(1/2), so raising to the 7th power gives (24/7)^(7/2)Therefore, a = 2*(24/7)^(7/2)Hmm, this seems a bit complicated. Let's check if that's correct.Wait, let's double-check the steps. So, we took the derivative, set it to zero, found a in terms of x. Then substituted back into f(x) to find the value at the critical point. Then set that value equal to 24, solved for x, then found a in terms of that x. But is there another way? Alternatively, maybe using the AM-GM inequality? Since we have two terms, 5(x + 1)^2 and a/(x + 1)^5. The function is the sum of these two terms, and we need to find the minimum a such that the sum is at least 24 for all x >= 0. AM-GM states that for non-negative numbers, the arithmetic mean is at least the geometric mean. Let's see if that applies here. Let me recall that AM-GM for two variables says that (p + q)/2 >= sqrt(pq), so p + q >= 2sqrt(pq). So maybe we can use that inequality here.Let me denote p = 5(x + 1)^2 and q = a/(x + 1)^5. Then f(x) = p + q. So, according to AM-GM, p + q >= 2*sqrt(pq). Therefore, f(x) >= 2*sqrt(5(x + 1)^2 * a/(x + 1)^5) = 2*sqrt(5a/(x + 1)^3). Hmm, but this gives a lower bound dependent on x. However, we need f(x) >= 24 for all x >= 0, so this lower bound must itself be at least 24. Therefore, 2*sqrt(5a/(x + 1)^3) >= 24. Let's solve for a:sqrt(5a/(x + 1)^3) >= 12Square both sides:5a/(x + 1)^3 >= 144Multiply both sides by (x + 1)^3/5:a >= (144/5)(x + 1)^3But this is a problem because x can vary. If a has to be greater than or equal to (144/5)(x + 1)^3 for all x >= 0, but as x increases, (x + 1)^3 increases without bound, so a would have to be infinite, which contradicts the problem statement that a is a positive constant. Therefore, this approach must be flawed, or perhaps AM-GM isn't the right tool here. Maybe because the two terms are not balanced in a way that allows AM-GM to give a useful bound here.Alternatively, let's go back to the calculus approach. We found that at the critical point, a = 2(x + 1)^7 and that the function evaluated at the critical point is 7(x + 1)^2 = 24. So solving 7(x + 1)^2 = 24 gives (x + 1)^2 = 24/7, so x + 1 = sqrt(24/7). Therefore, substituting back into a = 2(x + 1)^7 gives a = 2*(sqrt(24/7))^7. Let's compute this expression.First, note that sqrt(24/7) = (24/7)^(1/2), so raising this to the 7th power gives (24/7)^(7/2). Then multiply by 2:a = 2*(24/7)^(7/2)We can write 24 as 4*6, so 24 = 4*6, so (24/7) = (4*6)/7. Then:(24/7)^(7/2) = (4/1)^(7/2) * (6/7)^(7/2) = (2^2)^(7/2) * (6/7)^(7/2) = 2^7 * (6/7)^(7/2) = 128 * (6/7)^(7/2)Therefore, a = 2 * 128 * (6/7)^(7/2) = 256 * (6/7)^(7/2)But this seems complicated. Let me check if I can simplify (6/7)^(7/2):(6/7)^(7/2) = (6/7)^3 * (6/7)^(1/2) = (216/343) * sqrt(6/7)Therefore, a = 256 * (216/343) * sqrt(6/7)Compute 256 * 216: 256 * 200 = 51,200; 256 * 16 = 4,096; total = 55,296Then 55,296 / 343 = approximately 55,296 ÷ 343. Let's compute 343 * 160 = 54,880. Then 55,296 - 54,880 = 416. Then 416 ÷ 343 ≈ 1.213. So total is approximately 160 + 1.213 = 161.213. Then multiply by sqrt(6/7). sqrt(6/7) ≈ sqrt(0.857) ≈ 0.9258. So 161.213 * 0.9258 ≈ 149.3. So a ≈ 149.3. But maybe there's an exact value here. Let me check if the original expression can be simplified to an integer or a simpler fraction.Wait, maybe instead of using decimal approximations, I can express a in terms of radicals. Let's see:a = 2*(24/7)^(7/2)But 24/7 is 24 over 7, so (24/7)^(7/2) = 24^(7/2) / 7^(7/2) = (24^3 * sqrt(24)) / (7^3 * sqrt(7)) = (13,824 * 2*sqrt(6)) / (343 * sqrt(7)) )Wait, 24^3 is 24*24*24 = 576*24 = 13,824. sqrt(24) is 2*sqrt(6). Similarly, 7^3 is 343 and sqrt(7) is sqrt(7). So:24^(7/2) = 24^3 * sqrt(24) = 13,824 * 2*sqrt(6) = 27,648 * sqrt(6)7^(7/2) = 7^3 * sqrt(7) = 343 * sqrt(7)Therefore, (24/7)^(7/2) = (27,648 * sqrt(6)) / (343 * sqrt(7)) ) = (27,648 / 343) * sqrt(6/7)So then a = 2 * (27,648 / 343) * sqrt(6/7) = (55,296 / 343) * sqrt(6/7)Hmm, this is getting messy. Maybe there's a smarter way to write this. Let's check if 24 and 7 have some common factors. 24 is 4*6, 7 is prime. Maybe not. Alternatively, note that 24 = 3*8, but not sure.Alternatively, perhaps we can leave the answer as a = 2*(24/7)^(7/2), but this is not a simplified radical. However, maybe the problem expects an exact form, which would be a = 2*(24/7)^(7/2). Alternatively, perhaps we can express it as 2*(24)^{7/2}/7^{7/2} = 2*(24^3*sqrt(24))/7^3*sqrt(7)) = 2*(13824*2*sqrt(6))/(343*sqrt(7)) = 2*(27648*sqrt(6))/(343*sqrt(7)) = (55296 sqrt(6))/(343 sqrt(7)) = (55296/343)*sqrt(6/7). Then rationalize sqrt(6/7) as sqrt(42)/7, so:= (55296/343)*(sqrt(42)/7) = (55296 sqrt(42))/(343*7) = (55296 sqrt(42))/2401But 55296 divided by 2401. Let me compute 2401*23 = 2401*20 + 2401*3 = 48,020 + 7,203 = 55,223. Then 55,296 - 55,223 = 73. So 55296 = 2401*23 + 73. Therefore, 55296/2401 = 23 + 73/2401. So:a = (23 + 73/2401) sqrt(42) ≈ 23.0304 * 6.4807 ≈ 23.0304 * 6 + 23.0304 * 0.4807 ≈ 138.1824 + 11.077 ≈ 149.259. Which matches our previous approximation. So approximately 149.26. But the problem says "the minimum value of a", and since a must be an exact value, we need to present it in the form they want. However, looking back at the problem statement, it just says "the minimum value of a is...", so perhaps the answer is 256*(6/7)^(7/2). But maybe there's a simpler way.Wait, let me double-check my calculus approach. So, we found the critical point by setting the derivative to zero, found a in terms of x, substituted back into f(x) to find that at the critical point, the function is 7(x + 1)^2. Then set that equal to 24, solved for x + 1, then found a from there. That seems correct. Let me verify again.Given f(x) = 5(x + 1)^2 + a/(x + 1)^5. Take derivative: 10(x + 1) - 5a/(x + 1)^6. Set equal to zero: 10(x + 1) = 5a/(x + 1)^6 => 2(x + 1)^7 = a. So a = 2(x + 1)^7. Then substitute back into f(x):f(x) = 5(x + 1)^2 + [2(x + 1)^7]/(x + 1)^5 = 5(x + 1)^2 + 2(x + 1)^2 = 7(x + 1)^2. Then set 7(x + 1)^2 = 24 => (x + 1)^2 = 24/7 => x + 1 = sqrt(24/7). Therefore, a = 2*(sqrt(24/7))^7 = 2*(24/7)^(7/2). That's correct. So, that's the exact value of a. Then, maybe simplifying (24/7)^(7/2):24 = 4*6, so 24/7 = 4*6/7. Then (24/7)^(7/2) = (4/1)^(7/2)*(6/7)^(7/2) = (2^2)^(7/2)*(6/7)^(7/2) = 2^7*(6/7)^(7/2) = 128*(6/7)^(7/2). Then a = 2*128*(6/7)^(7/2) = 256*(6/7)^(7/2). But 6 and 7 are coprime, so this is as simplified as it can get. Alternatively, we can write this as 256*6^(7/2)/7^(7/2) = 256*sqrt(6^7)/sqrt(7^7) = 256*sqrt(6^6*6)/sqrt(7^6*7) = 256*(6^3*sqrt(6))/(7^3*sqrt(7)) = 256*(216*sqrt(6))/(343*sqrt(7)) = (256*216)/(343)*(sqrt(6)/sqrt(7)) = (55296/343)*sqrt(42)/7 = (55296/343)*(sqrt(42)/7) = (55296 sqrt(42))/(343*7) = 55296 sqrt(42)/2401. So, 55296 divided by 2401. Let's see if 2401 divides into 55296. 2401*23=55223, 55296-55223=73. So 55296=2401*23 +73. So, 55296/2401=23 +73/2401≈23.03. Then multiply by sqrt(42)≈6.4807. So, 23.03*6.4807≈149.26. So the minimal a is approximately 149.26. But the problem might want the exact value. So, the exact value is a=256*(6/7)^(7/2). But maybe there's a different way to write this.Alternatively, note that 6/7 is (6/7), so (6/7)^(7/2) = e^( (7/2) ln(6/7) ), but that's not helpful. Alternatively, rational exponents. Alternatively, since 256 is 2^8, and 6=2*3, so:256*(6/7)^(7/2) = 2^8 * (2*3/7)^(7/2) = 2^8 * 2^(7/2) * 3^(7/2) /7^(7/2) = 2^(8 + 7/2) * 3^(7/2)/7^(7/2) = 2^(23/2) * (3/7)^(7/2) = 2^(11.5) * (3/7)^3.5. Not helpful.Alternatively, writing exponents as fractions:a = 256 * 6^(7/2) / 7^(7/2) = 256 * (6^3 * sqrt(6)) / (7^3 * sqrt(7)) = 256 * 216 * sqrt(6) / (343 * sqrt(7)) = 256 * 216 / 343 * sqrt(42)/7 = (256*216)/(343*7) * sqrt(42) = (55296 / 2401) * sqrt(42). Which is the same as before. So, unless there's a simplification I'm missing, this is probably the simplest exact form.But maybe the problem expects an integer? Because 256 and 216 are powers of 2 and 6, but 343 is 7^3, so unless there is a cancellation, it's not an integer. Wait, 55296 and 2401: 55296 ÷ 49 = 55296 /49 = 1128.489, not an integer. So no, it's not an integer. Therefore, the minimal value of a is 256*(6/7)^(7/2), which is approximately 149.26. But the question says "the minimum value of a is". So, perhaps present the exact form. However, let me check if there's a different approach where the answer is an integer.Alternatively, maybe I made a mistake in the calculus approach. Let me check again.We have f'(x) = 10(x + 1) - 5a/(x + 1)^6. Setting this equal to zero:10(x + 1) = 5a/(x + 1)^6Multiply both sides by (x + 1)^6:10(x + 1)^7 = 5a => 2(x + 1)^7 = a. Correct.Then, substituting back into f(x):f(x) = 5(x + 1)^2 + a/(x + 1)^5 = 5(x + 1)^2 + 2(x + 1)^7/(x + 1)^5 = 5(x + 1)^2 + 2(x + 1)^2 = 7(x + 1)^2. Correct.Setting this equal to 24:7(x + 1)^2 = 24 => (x + 1)^2 = 24/7 => x + 1 = sqrt(24/7). Correct.Then, a = 2*(sqrt(24/7))^7 = 2*(24/7)^(7/2). Correct. So, this seems right.Therefore, the minimal a is 2*(24/7)^(7/2). To write this in terms of radicals:First, (24/7)^(7/2) = (24)^(7/2) / (7)^(7/2) = (24^3 * sqrt(24)) / (7^3 * sqrt(7)) = (13824 * 2*sqrt(6)) / (343 * sqrt(7)) ) = (27648 * sqrt(6)) / (343 * sqrt(7)) = multiplying numerator and denominator by sqrt(7) to rationalize the denominator:(27648 * sqrt(6) * sqrt(7)) / (343 * 7) = (27648 * sqrt(42)) / 2401Then, a = 2 * (27648 sqrt(42)/2401 ) = 55296 sqrt(42)/2401. So, this is the exact value.Alternatively, since 55296 divided by 2401 is approximately 23.03, and sqrt(42) is approximately 6.4807, so 23.03 * 6.4807 ≈ 149.26. So, the minimal a is approximately 149.26, but the exact value is 55296 sqrt(42)/2401. However, this is a rather complicated expression. Maybe the problem expects it written as 256*(6/7)^(7/2), which is another form. Let me check:256*(6/7)^(7/2) = 256 * 6^(7/2)/7^(7/2) = 256 * (6^3 * sqrt(6)) / (7^3 * sqrt(7)) ) = 256 * 216 * sqrt(6) / (343 * sqrt(7)) = same as before. So, same result.Alternatively, simplifying the exponents:6^(7/2) = 6^3 * 6^(1/2) = 216 * sqrt(6)7^(7/2) = 7^3 * 7^(1/2) = 343 * sqrt(7)So, 6^(7/2)/7^(7/2) = 216*sqrt(6)/343*sqrt(7) = 216/343 * sqrt(6/7) = 216/343 * sqrt(42)/7 = 216 sqrt(42)/2401Then, 256 * 216 sqrt(42)/2401 = (256*216)/2401 * sqrt(42). 256*216=55296. So, 55296 sqrt(42)/2401. So same as before.Therefore, the minimal a is 55296√42⁄2401. But 55296 and 2401 can be simplified? Let's check the GCD of 55296 and 2401. 2401 is 7^4, which is 49^2. 55296 ÷ 7: 55296 /7= 7899.428..., not integer. Therefore, the fraction 55296/2401 cannot be simplified further. Therefore, the exact value is 55296√42⁄2401. But this is quite a mouthful.Alternatively, perhaps the problem expects the answer in the form of 256*(6/7)^(7/2), which is more compact. But depending on the problem's expectation, they might prefer one form over the other. However, since the problem mentions the answer should be in a box and probably expects a simplified exact form, maybe we need to rationalize and present it as 55296√42⁄2401. Alternatively, maybe there's a different approach where the answer is a whole number.Wait, let's check by plugging x=0 into the original function. If x=0, then f(0)=5(1)^2 + a/(1)^5=5 +a. Since the function must be >=24 for all x>=0, including x=0. So 5 +a >=24 => a >=19. But if a=19, then the function at x=0 is 24, but maybe somewhere else it's lower. So, we need to ensure that the minimal value of the function over all x>=0 is 24, which occurs at the critical point we found. Therefore, a=19 is insufficient, as the function could dip below 24 for some x>0. Therefore, the minimal a must be larger than 19.Similarly, if we take x approaching infinity. As x→infty, (x +1)^2 grows like x^2, and a/(x +1)^5 tends to 0. Therefore, f(x) tends to infinity. So the function tends to infinity as x approaches infinity, so the minimum must occur somewhere finite, which is at the critical point we found. Hence, the minimal a is indeed 2*(24/7)^(7/2).Alternatively, perhaps there is a miscalculation here. Let me check again. Wait, in the step where we substituted a=2(x +1)^7 into f(x), let's confirm:f(x) =5(x +1)^2 + a/(x +1)^5. If a=2(x +1)^7, then a/(x +1)^5=2(x +1)^7/(x +1)^5=2(x +1)^2. So f(x)=5(x +1)^2 +2(x +1)^2=7(x +1)^2. Therefore, at the critical point, the function's value is 7(x +1)^2. Set that equal to 24, so (x +1)^2=24/7, so x +1= sqrt(24/7). Therefore, a=2*(sqrt(24/7))^7=2*(24/7)^(7/2). All steps correct.Alternatively, maybe we can write this in terms of exponents:(24/7)^(7/2) =24^(7/2)/7^(7/2)= (24^3 * sqrt(24))/ (7^3 * sqrt(7))= (13824 * 2*sqrt(6))/ (343 * sqrt(7))= 27648 * sqrt(6)/ (343 * sqrt(7)) =27648 * sqrt(42)/343 *7=27648 * sqrt(42)/2401. Then multiply by 2 gives 55296 sqrt(42)/2401. So that is exact. But perhaps the answer is written as 256*(6/7)^(7/2). Let me check if 55296/2401 =256*(216/343). Wait 256*(216/343)= (256*216)/343=55296/343, but we have 55296/2401. So no, 2401 is 343*7. So, 55296/2401= (55296/343)/7≈161.213/7≈23.03. So, it's not the same as 256*(6/7)^(7/2). Wait, but 256*(6/7)^(7/2)=256*(6^(7/2)/7^(7/2))=256*(6^3*sqrt(6))/7^3*sqrt(7))=256*(216*sqrt(6))/(343*sqrt(7))=256*216/343*sqrt(6/7)=55296/343*sqrt(42)/7=55296*sqrt(42)/2401. Yes, same as before. So, this is equivalent. So, either form is acceptable.However, given the problem's context, and since mathematical problems often prefer simplified radical form where possible, perhaps the answer is 256*(6/7)^(7/2). Alternatively, if the problem is from a calculus textbook, they might present it in the exponentiated form. But given that the answer is likely to be an integer, and 55296√42⁄2401 is not an integer, but the approximate value is ~149.26. Wait, but maybe the exact answer is 256*(6/7)^(7/2). However, to check if that's equivalent to the choices or if there's a miscalculation.Alternatively, maybe my entire approach is wrong. Let's think differently.Suppose we let t = x + 1, where t >=1 since x >=0. Then, the function becomes f(t) =5t^2 + a/t^5, for t >=1. We need to find the minimal a>0 such that f(t)>=24 for all t>=1.So, we can reframe the problem: find the minimal a>0 such that 5t^2 + a/t^5 >=24 for all t >=1.To find the minimal such a, we need to ensure that the minimal value of 5t^2 + a/t^5 over t >=1 is exactly 24. Therefore, the minimal a is such that the function 5t^2 + a/t^5 achieves 24 at its minimum point in t >=1.Therefore, we can use calculus here as well. Take derivative of f(t):f'(t) =10t -5a/t^6.Set to zero:10t =5a/t^6 => 2t^7 =a.Therefore, a=2t^7. Then, substitute back into f(t):f(t)=5t^2 + (2t^7)/t^5 =5t^2 +2t^2=7t^2.Set equal to24:7t^2=24 =>t^2=24/7 =>t= sqrt(24/7). Then a=2*(sqrt(24/7))^7=2*(24/7)^(7/2). Same result as before. So, regardless of substitution, the answer is the same. Therefore, this is the correct minimal a.Therefore, the minimal a is 2*(24/7)^(7/2). In LaTeX notation, this is written as 2 left( dfrac{24}{7} right)^{7/2}. However, since the problem might expect a numerical value or a simplified radical form, but given the exponents, it's likely they want the answer in the form with exponents. But maybe another approach.Alternatively, perhaps writing the exact value in terms of 24 and 7:Since 24 = 4*6, then 24/7 = 4*6 /7. Therefore,a =2*(4*6/7)^(7/2)=2*(4/1)^(7/2)*(6/7)^(7/2)=2*(2^2)^(7/2)*(6/7)^(7/2)=2*2^7*(6/7)^(7/2)=2^8*(6/7)^(7/2)=256*(6/7)^(7/2). So this is the same as before. Therefore, it's 256*(6/7)^3.5, but that's not helpful.Alternatively, note that (6/7)^(7/2) =6^(7/2)/7^(7/2)=6^3*sqrt(6)/7^3*sqrt(7)=216*sqrt(6)/(343*sqrt(7))=216*sqrt(42)/343*7=216*sqrt(42)/2401. Then, 256*216=55296, so a=55296*sqrt(42)/2401. So this is the exact value.But maybe the problem wants the answer in terms of 256*(6/7)^(7/2). Given the problem is likely from a calculus course, and considering that the answer is better presented in exponent form, I think the most appropriate exact answer is boxed{256 left( dfrac{6}{7} right)^{frac{7}{2}}}.Alternatively, if we want to rationalize and write it with a single radical:256*(6/7)^(7/2) =256*sqrt((6/7)^7)=256*sqrt(6^7/7^7)=256*sqrt(6^6*6 /7^6*7)=256*(6^3/7^3)*sqrt(6/7)=256*(216/343)*sqrt(42)/7=256*216*sqrt(42)/2401=55296*sqrt(42)/2401, which is another form. But since the problem says "the minimum value of a is", and given that multiple forms are acceptable, but perhaps the first form is more concise.Alternatively, check if 256*(6/7)^(7/2) can be written as 256*(6^3.5)/7^3.5, but not sure. Alternatively, convert the exponent 7/2 into a radical:(6/7)^(7/2) = sqrt((6/7)^7) = sqrt(6^7/7^7) = sqrt(279,936 / 823,543) ≈ sqrt(0.3398) ≈ 0.583. Then 256*0.583≈149.25, which matches our approximate value.Given that the problem might expect an exact answer, but the form is not standard, I think the most precise answer is boxed{256 left( dfrac{6}{7} right)^{frac{7}{2}}}.Alternatively, maybe the answer is expected to be simplified with numbers. Wait, if we compute 6^7=279,936 and 7^7=823,543. Then (6/7)^7=279,936/823,543. Then sqrt(279,936/823,543)=sqrt(279936)/sqrt(823543)=529.1/907.55≈0.583. So, a=256*0.583≈149.25. But since it's exact value, we can't write the approximate. So, the exact answer is boxed{256 left( dfrac{6}{7} right)^{frac{7}{2}}}.Alternatively, another way to present it could be:256 multiplied by (6 over 7) raised to the power of 3.5, but in LaTeX, fractions are preferred for exponents.Therefore, the exact minimal value of a is 256*(6/7)^(7/2), which is the boxed answer.Final AnswerThe minimum value of ( a ) is boxed{256 left( dfrac{6}{7} right)^{frac{7}{2}}}.</think>To find the minimum value of ( a ) such that ( f(x) = 5(x+1)^2 + frac{a}{(x+1)^5} ) (where ( a > 0 )) is at least 24 for all ( x geq 0 ), we start by analyzing the critical points of the function.1. Compute the derivative of ( f(x) ): [ f'(x) = 10(x + 1) - frac{5a}{(x + 1)^6} ]2. Set the derivative to zero to find critical points: [ 10(x + 1) = frac{5a}{(x + 1)^6} ] Simplifying this, we get: [ 2(x + 1)^7 = a ]3. Substitute ( a = 2(x + 1)^7 ) back into the original function: [ f(x) = 5(x + 1)^2 + frac{2(x + 1)^7}{(x + 1)^5} = 5(x + 1)^2 + 2(x + 1)^2 = 7(x + 1)^2 ]4. Set the function value at the critical point to 24: [ 7(x + 1)^2 = 24 implies (x + 1)^2 = frac{24}{7} implies x + 1 = sqrt{frac{24}{7}} ]5. Find ( a ) using the critical point: [ a = 2 left( sqrt{frac{24}{7}} right)^7 = 2 left( frac{24}{7} right)^{7/2} ]6. Simplify the expression: [ a = 2 left( frac{24}{7} right)^{7/2} = 256 left( frac{6}{7} right)^{7/2} ]Thus, the minimum value of ( a ) is boxed{256 left( dfrac{6}{7} right)^{frac{7}{2}}}.]

🔑:9. 2 sqrt{left(frac{24}{7}right)^{7}}. Since f(x)=underbrace{(x+1)^{2}+cdots+(x+1)^{2}}_{1 uparrow}+frac{a}{2(x+1)^{5}}+frac{a}{2(x+1)^{5}} geqslant 7 cdot sqrt[7]{frac{a^{2}}{4}}( when and only when (x+1)^{2}=frac{a}{2(x+1)^{3}}, the equality holds ).Thus, 7 cdot sqrt[7]{frac{a^{2}}{4}} geqslant 24 Rightarrow a geqslant 2 sqrt{left(frac{24}{7}right)^{7}}.On the other hand, when a=2 sqrt{left(frac{24}{7}right)^{7}}, x=sqrt{frac{24}{7}}-1, f(x)=24.

❓:Which of the following equations is a quadratic equation?A: x^{2}+y=11B: x^2-frac{1}{x}=1C: x^{2}-13=0D: 2x+1=0

🧠:[<think>Okay, so I need to figure out which of the given options is a quadratic equation. Let me start by recalling what a quadratic equation is. From what I remember, a quadratic equation is a second-degree polynomial equation in one variable. That means it should have the form ax² + bx + c = 0, where a, b, and c are constants, and a is not zero. The key points here are that it's a polynomial (so no variables in denominators or negative exponents), it's in one variable, and the highest exponent on the variable is 2.Let me look at each option one by one.Starting with option A: x² + y = 11. Hmm, this equation has two variables, x and y. Quadratic equations are supposed to be in one variable, right? So if there's another variable present, like y here, does that disqualify it? I think so. Because even though there's an x² term, since there's another variable y, this is more like a quadratic in two variables, which would make it a quadratic form, but not a quadratic equation in one variable. So probably option A isn't the answer.Moving on to option B: x² - 1/x = 1. Let me rewrite this to check. If I subtract 1 from both sides, it becomes x² - 1/x - 1 = 0. The term 1/x is the same as x^(-1), which means this equation has a term with a negative exponent. Quadratic equations are polynomials, and polynomials don't have variables with negative exponents. So this equation isn't a polynomial equation, hence not a quadratic equation. So option B is out.Option C: x² - 13 = 0. Let's see. This equation has only the variable x, and the highest power of x is 2. There are no other terms with x, just the x² term and a constant. But even so, this fits the form ax² + bx + c = 0 where b is 0. So it's a quadratic equation. This seems like a valid candidate.Option D: 2x + 1 = 0. Here, the highest power of x is 1. So this is a linear equation, not a quadratic one. Therefore, option D is not the answer.Let me double-check each option to make sure I didn't overlook something.For option A again: two variables, so even if you solve for y, it becomes y = -x² + 11, which is a quadratic function in terms of x, but the equation itself is not a quadratic equation in one variable. Quadratic equations need to be equations where the highest degree term is 2 and in a single variable. So A is out.Option B: Negative exponent, which makes it not a polynomial. So even if you multiply through by x to eliminate the denominator, you would get x³ - 1 - x = 0, which is a cubic equation (degree 3). So that's why it's not quadratic. So B is not quadratic.Option C: Definitely quadratic. Simple as that. x squared minus 13 equals zero. No other terms, just the quadratic term and constant. So that's quadratic.Option D: Linear. Degree 1. So no.Therefore, the correct answer should be option C.Final Answerboxed{C}</think>To determine which of the given equations is a quadratic equation, we recall that a quadratic equation is a second-degree polynomial equation in one variable, having the form (ax^2 + bx + c = 0) where (a neq 0).- Option A: (x^2 + y = 11) This equation contains two variables (x) and (y). Quadratic equations must be in one variable, so this is not a quadratic equation.- Option B: (x^2 - frac{1}{x} = 1) Rewriting this equation, we get a term (frac{1}{x}) which is equivalent to (x^{-1}). This introduces a negative exponent, making it a non-polynomial equation. Thus, this is not a quadratic equation.- Option C: (x^2 - 13 = 0) This equation is a polynomial in one variable (x) with the highest degree term (x^2). It fits the form (ax^2 + bx + c = 0) where (a = 1), (b = 0), and (c = -13). Therefore, this is a quadratic equation.- Option D: (2x + 1 = 0) This is a linear equation (degree 1) in one variable, not a quadratic equation.The correct answer is (boxed{C}).,<think>Okay, let's see. The question is asking which of the given equations is a quadratic equation. The options are A, B, C, and D. Hmm, I need to remember what defines a quadratic equation. From what I recall, a quadratic equation is a second-degree polynomial equation in one variable. That means it should have the form ax² + bx + c = 0, where a, b, c are constants and a is not zero. The key points here are that it should have one variable, the highest exponent of that variable should be 2, and there should be no negative exponents or variables in denominators because that would make it not a polynomial.Let's look at each option one by one.Starting with option A: x² + y = 11. Wait, this equation has two variables, x and y. Quadratic equations are supposed to be in one variable, right? So even though there's an x squared term, having another variable y might disqualify it. Unless the question allows multiple variables, but typically quadratic equations are in one variable. So maybe A isn't the answer. I'll keep that in mind but check the others.Option B: x² - 1/x = 1. Hmm, here we have an x squared term, but there's also a 1/x, which is the same as x to the power of -1. Since quadratic equations are polynomials, they can't have negative exponents. So this term with 1/x would make it not a polynomial equation. Therefore, B probably isn't a quadratic equation either.Option C: x² - 13 = 0. This looks more straightforward. It has x squared as the highest power, no other variables, and no negative exponents. So if we rearrange it, it's x² + 0x - 13 = 0, which fits the standard quadratic equation form ax² + bx + c = 0 where a=1, b=0, and c=-13. So this seems like a quadratic equation.Option D: 2x + 1 = 0. This is a linear equation because the highest power of x is 1. So definitely not quadratic.Let me just double-check each one again to make sure I didn't miss anything.A: Two variables, x and y. If the problem is considering equations in two variables, then technically this is a quadratic equation in two variables, but usually when someone refers to a quadratic equation without specifying, they mean in one variable. So A is quadratic in x but also includes y, which isn't part of the standard definition.B: The term 1/x is equivalent to x^(-1), which is a negative exponent. Polynomials don't allow negative exponents, so B isn't a polynomial equation, hence not quadratic.C: Only x squared term and a constant, no other terms. This is a pure quadratic equation in one variable.D: Linear equation, so no.Therefore, the answer should be option C.Final Answerboxed{C}</think>To determine which equation is a quadratic equation, we need to check if it is a second-degree polynomial equation in one variable. A quadratic equation should have the form (ax^2 + bx + c = 0) where (a neq 0), and there should be no negative exponents or variables in the denominator.- Option A: (x^2 + y = 11) This equation has two variables (x) and (y). Quadratic equations are typically defined for a single variable, so this is not a quadratic equation.- Option B: (x^2 - frac{1}{x} = 1) The term (frac{1}{x}) is equivalent to (x^{-1}), which is a negative exponent. Since polynomials do not allow negative exponents, this is not a quadratic equation.- Option C: (x^2 - 13 = 0) This equation is a second-degree polynomial in one variable (x). It can be rewritten as (x^2 + 0x - 13 = 0), which fits the form of a quadratic equation.- Option D: (2x + 1 = 0) This is a linear equation (degree 1), not a quadratic equation.Thus, the correct answer is (boxed{C}).]

🔑:To determine which of the given equations is a quadratic equation, we analyze each option step by step:- For option A: x^{2}+y=11, we observe that this equation involves two variables, x and y. A quadratic equation typically has one variable squared, and although x^2 fits this criterion, the presence of a second variable, y, means this is not a standard quadratic equation in one variable. Therefore, option A does not meet the requirements for a quadratic equation.- For option B: x^2-frac{1}{x}=1, this equation involves x^2 and also a term with x in the denominator, frac{1}{x}. This makes the equation a fractional equation rather than a standard quadratic equation, which should be in the form ax^2 + bx + c = 0 without fractions. Thus, option B does not meet the requirements for a quadratic equation.- For option C: x^{2}-13=0, this equation is in the form ax^2 + bx + c = 0 where a=1, b=0, and c=-13. This fits the definition of a quadratic equation perfectly, as it involves a single variable x, squared, and no higher powers or fractions. Therefore, option C meets the requirements for a quadratic equation.- For option D: 2x+1=0, this equation is linear, not quadratic, because it involves x to the first power only. A quadratic equation would require an x^2 term. Therefore, option D does not meet the requirements for a quadratic equation.Given the analysis above, the correct answer is option C, as it is the only equation that is a quadratic equation with one variable.Therefore, the correct answer is: boxed{C}.

Released under the MIT License.

has loaded